Sei sulla pagina 1di 117

Set 3 Block 1:

Exam Title :
Geography & ...
Email : misrapulkit@yahoo.in
Contact :

QUESTION 1.
Which of the following pair(s) given below is/are correctly matched?

1. Saddle Peak --- North Andaman


2. Mount Thuiller --- South Andaman
3. Mount Koyob --- Great Nicobar

Select the correct code given below

a) 1 only
b) 1 and 2 only
c) 2 and 3 only
d) 3 only
Correct Answer: A
Your Answer: Unanswered
Explanation

Solution (a)

Some important mountain peaks in Andaman and Nicobar islands are Saddle peak (North
Andaman – 738 m), Mount Diavolo (Middle Andaman – 515 m), Mount Koyob (South Andaman –
460 m) and Mount Thuiller (Great Nicobar – 642 m).

QUESTION 2.
Consider the following statements regarding the Coastal Plains of India

1. As compared to eastern coastal plain, the western coastal plain is broader.


2. The western Coastal plain is an emergent coast.
3. There are well developed deltas in the eastern coastal plains.

Which of the above statement(s) is/are correct?

a) 1 and 2 only
b) 1 only
c) 3 only
d) 1, 2 and 3
Correct Answer: C
Your Answer: Unanswered
Explanation

Solution (c)

As compared to the western coastal plain, the eastern coastal plain is broader and is an
example of an emergent coast. There are well developed deltas here, formed by the rivers
flowing eastward in to the Bay of Bengal. These include the deltas of the Mahanadi, the

Godavari, the Krishna and the Kaveri. Because of its emergent nature, it has less number of

ports and harbours.

IASbaba
Web: http://ilp.iasbaba.com/ Score:
Email: ilp@iasbaba.com 0.00 / 200
Page 1
Set 3 Block 1:
Exam Title :
Geography & ...
Email : misrapulkit@yahoo.in
Contact :

QUESTION 3.
Consider the following statements regarding the Indian Physiography of India:

1. The Peninsular Plateau is one of the oldest and the most stable landmass of India.
2. The northern plains are formed by the alluvial deposited brought by the rivers.

Which of the above statement(s) is/are correct?

a) 1 only
b) 2 only
c) 1 and 2
d) Neither 1 nor 2
Correct Answer: C
Your Answer: Unanswered
Explanation

Solution (c)

The Peninsular India is made up of a series of patland plateaus such as the Hazaribagh plateau,
the Palamu plateau, the Ranchi plateau, the Malwa plateau, the Coimbatore plateau and the
Karnataka plateau, etc. This is one of the oldest and the most stable landmass of India.

The northern plains are formed by the alluvial deposits brought by the rivers – the Indus, the
Ganga and the Brahmaputra. These plains extend approximately 3,200 km from the east to the
west. The average width of these plains varies between150-300 km. The maximum depth of
alluvium deposits varies between 1,000-2,000 m.

QUESTION 4.
The 8 degree channel separate which one the following?

a) Lakshdweep and Minicoy Islands


b) Maldives and Minicoy Islands
c) Andaman and Nicobar Islands
d) Indira point and Indonesia
Correct Answer: B
Your Answer: Unanswered
Explanation

Solution (b)

1) 6 degree or 6° channel – Separates Indira point and Indonesia

2) 8 degree or 8° channel – Seperates Maldives and Minicoy

3) 9 degree or 9° channel – Separates Lakshdweep and Minicoy islands

4) 10 degree or 10° channels – Separates Andaman and Nicobar channel

IASbaba
Web: http://ilp.iasbaba.com/ Score:
Email: ilp@iasbaba.com 0.00 / 200
Page 2
Set 3 Block 1:
Exam Title :
Geography & ...
Email : misrapulkit@yahoo.in
Contact :

QUESTION 5.
Which Indian state is also known as the ‘Molasses Basin’?

a) Manipur
b) Tripura
c) Nagaland
d) Mizoram
Correct Answer: D
Your Answer: Unanswered
Explanation

Solution (d)

Mizoram is known as LAND OF ROLLING MOUNTAINS , i.e i t has huge number of


mountains formation of most mountains are accompanied by formation of foreland basin or in
simple terms valley type depression which runs parallel to mountains. These depression get
accumulated with unconsolidated deposits known as Molasses basin.

QUESTION 6.
Which of the following are aligned correctly from north to south?

a) Karakoram – Zaskar – Ladakh – Pir Panjal


b) Zaskar – Karakoram – Ladakh – Pir Panjal
c) Karakoram – Ladakh – Zaskar – Pir Panjal
d) Karakoram – Zaskar – Pir Panjal – Ladakh
Correct Answer: C
Your Answer: Unanswered
Explanation

Solution (c)

IASbaba
Web: http://ilp.iasbaba.com/ Score:
Email: ilp@iasbaba.com 0.00 / 200
Page 3
Set 3 Block 1:
Exam Title :
Geography & ...
Email : misrapulkit@yahoo.in
Contact :

QUESTION 7.
Consider the following statements regarding the Islands of India

1. The Andaman Nicobar islands and Lakshadweep islands are originated from volcanic
eruptions.
2. New Moore Island in the Bay of Bengal emerged aftermath of the Bhola cyclone in 1970.

Which of the above statement(s) is/are correct?

a) 1 only
b) 2 only
c) 1 and 2
d) Neither 1 nor 2
Correct Answer: B
Your Answer: Unanswered
Explanation

Solution (b)

IASbaba
Web: http://ilp.iasbaba.com/ Score:
Email: ilp@iasbaba.com 0.00 / 200
Page 4
Set 3 Block 1:
Exam Title :
Geography & ...
Email : misrapulkit@yahoo.in
Contact :

The Andaman Nicobar islands originated from Volcanic eruptions whereas Lakshadweep isl
ands are originated from Coral Reefs.

New Moore Island is a small uninhabited offshore sandbar landform {Marine Landforms} in
the Bay of Bengal, off the coast of the Ganges-Brahmaputra Delta region. It emerged in the Bay
of Bengal in the aftermath of the Bhola cyclone in 1970 . It keeps on emerging and
disappearing. Although the island was uninhabited and there were no permanent settlements or
stations located on it, both India and Bangladesh claimed sovereignty over it because of
speculation over the existence of oil and natural gas in the region .

QUESTION 8.
Consider the following statements regarding the Coastal Plain of India

1. The river flowing through the western coastal plain do not form any delta
2. The Malabar Coast has a distinguishing feature in the form of kayals .
3. The famous Nehru Trophy Vallamkali is held in Punnamada Kayal in Kerala.

Which of the above statement(s) is/are correct?

a) 1 only
b) 1 and 2 only
c) 2 and 3
d) 1 ,2 and 3
Correct Answer: D

IASbaba
Web: http://ilp.iasbaba.com/ Score:
Email: ilp@iasbaba.com 0.00 / 200
Page 5
Set 3 Block 1:
Exam Title :
Geography & ...
Email : misrapulkit@yahoo.in
Contact :
Your Answer: Unanswered
Explanation

Solution (d)

The western coastal plains are narrow in the middle and get broader towards north and south.
The rivers flowing through this coastal plain do not form any delta. The Malabar coast has got
certain distinguishing features in the form of ‘Kayals’ (backwaters), which are used for fishing,
inland navigation and also due to its special attraction for tourists. Every year the famous Nehr
u Trophy Vallamkali (boat race) is held in Punnamada Kayal in Kerala.

QUESTION 9.
Consider the following statements regarding the Northern Plains.

1. Tarai is the northern most layers of the plains, south of the Siwaliks from west to east.
2. Bhabhar lies South of Tarai with its width increasing from west to east.
3. Bhabhar is not suitable for cultivation whereas Tarai belt is well known for cultivation of
Sugar, rice, pulses. etc.

Which of the above statement(s) is/are correct?

a) 1 and 2 only
b) 2 and 3 only
c) 1 only
d) 3 only
Correct Answer: D
Your Answer: Unanswered
Explanation

Solution (d)

Source: VAN Geography

The Bhabar Plains:

IASbaba
Web: http://ilp.iasbaba.com/ Score:
Email: ilp@iasbaba.com 0.00 / 200
Page 6
Set 3 Block 1:
Exam Title :
Geography & ...
Email : misrapulkit@yahoo.in
Contact :

· This is the northern most layer of the plains. It lies South of the Siwaliks from west to east.

· The width of this region is not uniform and it is wider in west than in East. The complete
stretch is 8-15km wide.

· As the rivers enter suddenly into the plains from Himalayas, the deposition starts and heavier
particles are deposited. Hence the sediments consist of gravel and unassorted particles. The
porosity of this region is so high that most of the small streams (Chos) disappear in the bhabar
tract.

· It is not suitable for cultivation. Only big tress can thrive in the region.

The Tarai Tract:

IASbaba
Web: http://ilp.iasbaba.com/ Score:
Email: ilp@iasbaba.com 0.00 / 200
Page 7
Set 3 Block 1:
Exam Title :
Geography & ...
Email : misrapulkit@yahoo.in
Contact :

Tarai Tract

· Tarai lies South of Bhabar.

· It is 15-30km wide with its width increasing from west to east. (Note: this is opposite from the
Bhabar plains).

· This is a zone of excessive dampness, thick forests, rich wild life and malarial climate.

· This zone is formed as the rivers which got submerged in Bhabar plains reemerge in this
region.

· In most of the northen states, from Haryana to Bihar, the Tarai forests have been cleared and
plains are used for agriculture now.

The Tarai belt is known for the cultivation of Sugarcane, rice, wheat, maize, oil seeds, pulses
and fodder.

QUESTION 10.
‘ Aniamudi ’ is the highest peak of which of the following hill range?

a) Niligiri hills

IASbaba
Web: http://ilp.iasbaba.com/ Score:
Email: ilp@iasbaba.com 0.00 / 200
Page 8
Set 3 Block 1:
Exam Title :
Geography & ...
Email : misrapulkit@yahoo.in
Contact :

b) Nallamala hills
c) Anaimalai hills
d) Cardamom hills
Correct Answer: C
Your Answer: Unanswered
Explanation

Solution (c)

‘Anaimudi’ (2,695 m), the highest peak of Peninsular plateau is located on the Anaimalai hills of
the Western Ghats followed by Dodabetta (2,637 m) on the Nilgiri hills.

QUESTION 11.
On the basis of prominent relief features consider the following statements on Central
highlands

1. The Satpura range lies to the eastern most end of the Central Highlands
2. The Central Highlands are bounded to the North-West by the Aravali range.
3. Central Highlands are wider in the West but narrower in the East.

Which of the following statements(s) is/are correct?

a) 1 and 2 only
b) 2 and 3 only
c) 1 only
d) 3 only
Correct Answer: B
Your Answer: Unanswered
Explanation

Solution (b)

The Central Highlands

They are bounded to the North-West by the Aravali range . The Satpura range is
formed by a series of scarped plateaus on the south , generally at an elevation varying
between 600-900 m above the mean sea level. This forms the northernmost boundary of the
Deccan plateau. It is a classic example of the relict mountains which are highly denuded and
form discontinuous ranges. The extension of the Peninsular plateau can be seen as far as
Jaisalmer in the West, where it has been covered by the longitudinal sand ridges and crescent-
shaped sand dunes called barchans . This region has undergone metamorphic processes in its
geological history, which can be corroborated by the presence of metamorphic rocks such as
marble, slate, gneiss, etc.

They are wider in the West but narrower in the East.

IASbaba
Web: http://ilp.iasbaba.com/ Score:
Email: ilp@iasbaba.com 0.00 / 200
Page 9
Set 3 Block 1:
Exam Title :
Geography & ...
Email : misrapulkit@yahoo.in
Contact :

IASbaba
Web: http://ilp.iasbaba.com/ Score:
Email: ilp@iasbaba.com 0.00 / 200
Page 10
Set 3 Block 1:
Exam Title :
Geography & ...
Email : misrapulkit@yahoo.in
Contact :

IASbaba
Web: http://ilp.iasbaba.com/ Score:
Email: ilp@iasbaba.com 0.00 / 200
Page 11
Set 3 Block 1:
Exam Title :
Geography & ...
Email : misrapulkit@yahoo.in
Contact :

QUESTION 12.
Why India’s North-South distance is more than the East-West, though both latitudinal and
longitudinal extent is about 30 degree?

a) Because the longitudes remains same whereas latitudes distances increases as it goes
further.
b) Because the longitudes distance decreases as it goes further whereas latitudes remains
same.
c) Because the longitudes remains same whereas latitudes distances decreases as it goes
further
d) Because the longitudes distance increases as it goes further whereas latitudes remains
same.
Correct Answer: B
Your Answer: Unanswered
Explanation

Solution (b)

India ’s North-South distance is more than the East-West, though both latidunal and
longitudinal extent is about 30 degree. This is because the distance between the longitudes
decreases near the poles whereas the distances between latitudes remain same everywhere.

IASbaba
Web: http://ilp.iasbaba.com/ Score:
Email: ilp@iasbaba.com 0.00 / 200
Page 12
Set 3 Block 1:
Exam Title :
Geography & ...
Email : misrapulkit@yahoo.in
Contact :
QUESTION 13.
Consider the following statements about the Gulfs of India.

1. The Gulf of Mannar is at southeastern tip of India in the Malabar Coast Region.

2. The Gulf of Khambhat is located between the two states Gujarat and Maharashtra

3. The Gulf of Kutch region is prone to severe daily tides.

Which of the above statement(s) is/are correct?

a) 1 and 2 only
b) 2 and 3 only
c) 1 only
d) 3 only
Correct Answer: D
Your Answer: Unanswered
Explanation

Solution (d)

Gulf of Mannar is located between the southeastern tip of India and the west coast of Sri
Lanka in the Coromandel Coast Region and forms a part of the Laccadive Sea in the

Indian Ocean .

Gulf of Khambhat or Gulf of Cambay is located in the state of Gujarat , along the west coast
of India. It can be defined as an inlet of the Arabian Sea. It has a length of about 130 kilometers
miles and is fed by the rivers namely, Narmada, Tapti, Mahi and Sabarmati.

Kathiawar peninsula to the west is separated from the eastern part of Gujarat by the Gulf of
Khambhat.

Gulf of Kutch is located in the state of Gujarat, along the west coast of India. It is defined as
an inlet of the Arabian Sea. It is about 99 miles in length and has maximum depth of about 401
feet. The gulf region is prone to severe daily tides, which makes it suitable for tidal
energy generation. It also functions to separate Kutch and Kathiawar peninsula regions of
Gujarat.

QUESTION 14.
Consider the following statements regarding the Eastern Hills and mountains

1. Khasi hills and Mizo hills are part of Patkai Range.

2. Khasi hills are also known as Lushai hills in local language.

3. Phawngpui or ‘Blue Mountain’ is the highest point of Mizo hills.

IASbaba
Web: http://ilp.iasbaba.com/ Score:
Email: ilp@iasbaba.com 0.00 / 200
Page 13
Set 3 Block 1:
Exam Title :
Geography & ...
Email : misrapulkit@yahoo.in
Contact :

Which of the above statement(s) is/are correct?

a) 1 and 2 only
b) 2 and 3 only
c) 1 and 3 only
d) 1, 2 and 3
Correct Answer: C
Your Answer: Unanswered
Explanation

Solution (c)

The Eastern Hills and Mountains are part of the Himalayan mountain system having their
general alignment from the north to the south direction. They are known by different local
names. In the north, they are known as Patkai Bum, Naga hills, the Manipur hills and in the
south as Mizo or Lushai hills . These are low hills, inhabited by numerous tribal groups
practising Jhum cultivation.

Khasi hills and Mizo hills are part of Patkai Range.

Phawngpui or ‘Blue Mountain’ is the highest point of Mizo hills.

Both are factual statements and must be memorized.

IASbaba
Web: http://ilp.iasbaba.com/ Score:
Email: ilp@iasbaba.com 0.00 / 200
Page 14
Set 3 Block 1:
Exam Title :
Geography & ...
Email : misrapulkit@yahoo.in
Contact :

QUESTION 15.
When you travel in Himalayas, you will see the following:

1. Deep gorges
2. U-turn river courses
3. Parallel mountain ranges
4. Steep gradients causing land-sliding

Which of the above can be said to be the evidences for Himalayas being young fold mountains?

IASbaba
Web: http://ilp.iasbaba.com/ Score:
Email: ilp@iasbaba.com 0.00 / 200
Page 15
Set 3 Block 1:
Exam Title :
Geography & ...
Email : misrapulkit@yahoo.in
Contact :

a) 1 and 2 only
b) 1, 2 and 4 only
c) 3 and 4 only
d) 1, 2, 3 and 4
Correct Answer: D
Your Answer: Unanswered
Explanation

Solution (d)

The Himalayas along with other Peninsular mountains are young, weak and flexible in their
geological structure unlike the rigid and stable Peninsular Block. Consequently, they are still
subjected to the interplay of exogenic and endogenic forces, resulting in the development of
faults, folds and thrust plains. These mountains are tectonic in origin, dissected by fast-flowing
rivers which are in their youthful stage. Various landforms like gorges, V-shaped valleys, rapids,
waterfalls, etc. are indicative of this stage.

QUESTION 16.
Consider the following statements regarding the Karewas formation:

1. ‘ Karewas ’ in Kashmiri language refer to the lake deposits, found in the flat-topped
terraces of the Kashmir valley and on the flanks of the Zaskar range
2. These deposits consist of clays, silts and sands, these deposits also show evidence of
glaciation.
3. Karewas are famous for the cultivation of Zafran , a local variety of saffron.

Which of the following statements(s) is/are correct?

a) 1 and 2 only
b) 2 and 3 only
c) 1 and 3 only
d) 1, 2 and 3
Correct Answer: B
Your Answer: Unanswered
Explanation

Solution (b)

The Kashmir Himalayas are also famous for Karewa formations .

· ‘Karewas’ in Kashmiri language refer to the lake deposits, found in the flat-topped terraces of
the Kashmir valley and on the flanks of the Pir Panjal range.

· These deposits consist of clays, silts and sands, these deposits also show evidence of
glaciation.

· The occurrence of tilted beds of Karewas at the altitudes of 1500-1800m on the flanks of the
Pir Panjal strongly suggests that the Himalayas were in process of uplift as late as Pliocene and
Pleistocene

· Karewas are famous for the cultivation of Zafran , a local variety of saffron.

IASbaba
Web: http://ilp.iasbaba.com/ Score:
Email: ilp@iasbaba.com 0.00 / 200
Page 16
Set 3 Block 1:
Exam Title :
Geography & ...
Email : misrapulkit@yahoo.in
Contact :

QUESTION 17.
Consider the following statements regarding the passes of India.

1. Jalep La (Pass) is at the tri-junction of India-China-Bhutan.


2. The Palghat gap is located at the southern end of the Niligiri hills in South India
3. Zoji La (Pass) is in the Zaskar range

Which of the following statements(s) is/are correct?

a) 1 and 2 only
b) 2 and 3 only
c) 1 and 3 only
d) 1, 2 and 3
Correct Answer: D
Your Answer: Unanswered
Explanation

Solution (d)

1. Zoji La (Pass) – It is in the Zaskar range of Jammu and Kashmir. The land route from
Srinagar to Leh goes through this pass.
2. Jalep La (Pass) – At the tri-junction of India- China-Bhutan. The Teesta river has created
this pass.
3. The Palghat gap is located at the southern end of the Niligiri hills in South India. Locate
d between the Nilgiri Hills to the north and the Anaimalai Hills to the south, it is about 20
miles (32 km) wide and straddles the Kerala–Tamil Nadu border, serving as a major
communication route between those two states. Highways and rail lines through the gap
connect Palghat in Kerala with Coimbatore and Pollachi in Tamil Nadu . Palghat Gap also
influences southern India’s climate; the wet southwest monsoons as well as
storms from the Bay of Bengal cross the mountains through the opening.

QUESTION 18.
Consider the following statements about the Western and Eastern Himalayas of India.

1. The Western Himalayas are higher with sudden steep slope whereas Eastern Himalayas
are lower with gradual slope.
2. The Western Himalayas are located on higher latitude whereas the Eastern Himalayas are
located on lower latitude.

Which of the above statement(s) is/are correct?

a) 1 only
b) 2 only
c) 1 and 2
d) Neither 1 nor 2
Correct Answer: B
Your Answer: Unanswered
Explanation

IASbaba
Web: http://ilp.iasbaba.com/ Score:
Email: ilp@iasbaba.com 0.00 / 200
Page 17
Set 3 Block 1:
Exam Title :
Geography & ...
Email : misrapulkit@yahoo.in
Contact :

Solution (b)

Western Himalayas Eastern Himalayas

1. Lower and gradual slope. Hence, the 1. Higher and steep-sudden slope. That is
higher peaks in this part are farther from why two of the highest peaks of Himalayas,
the plains and a number of ranges lie Mt. Everest (in Nepal) and Kanchenjunga
between the plains and high peak are not very far from the plains.

2. Located on higher latitude, therefore


2. Located on lower latitude, therefore
colder. As a result, the snowline in the
warmer. Therefore, snowline is at a higher
Western Himalayas is at a lower altitude
altitude.
than in the Eastern Himalayas

3. From Indus to Kali river 3. From Kali river to Brahmaputra river

4. Peaks: Nanga Parbat, Nanda Devi, 4. Peaks: Everest, Makalu, Annapurna,


Badrinath Dhaulagiri.

QUESTION 19.
Consider the following statements regarding Hill Ranges of Peninsular India.

1. Most of the hills in the peninsular region are of relict type.


2. Vindhya Range acts as a watershed between the Ganga system and the river systems of
South India.
3. Amarkantak is the highest peak of Satpura range

Which of the above statements are correct?

a) 1 and 2 only
b) 2 and 3 only
c) 1 and 3 only
d) 1, 2 and 3
Correct Answer: A
Your Answer: Unanswered
Explanation

Solution (a)

· Most of the hills in the peninsular region are of the relict type (residual hills).

· They are the remnants of the hills and horsts formed many million years ago (horst: uplifted
block; graben: subsided block).

· H ills like Nilgiri, Palkonda, Parasnath and Rajmahal and Mountains like the Aravalli, the
Vindhya, and the Satpura are some of the examples of Relict Mountains in India.

IASbaba
Web: http://ilp.iasbaba.com/ Score:
Email: ilp@iasbaba.com 0.00 / 200
Page 18
Set 3 Block 1:
Exam Title :
Geography & ...
Email : misrapulkit@yahoo.in
Contact :

· Vidhayan range acts as a watershed between the Ganga system and the river systems of south
India. In this range the rivers like Chambal, Betwa and Ken rise within 30 km of the
Narmada.

· Parts of the Satpura have been folded and upheaved. They are regarded as structural uplift or
‘horst’ . Dhupgarh (1,350 m) near Pachmarhi on Mahadev Hills is the highest peak. Amar
kantak (1,127 m) is another important peak.

QUESTION 20.
Three important rivers of the Indian subcontinent have their sources near the Mansarovar Lake
in the Great Himalayas. Which are these rivers?

a) Jhelum, Sutlej and Yamuna


b) Brahmaputra, Indus and Sutlej
c) Brahmaputra, Sutlej and Yamuna
d) Indus, Jhelum and Sutlej
Correct Answer: B
Your Answer: Unanswered
Explanation

Solution (b)

Manasarovar is the origin of four main rivers namely, Sutlej river, Indus river, Brahmaputra
river and Karnali river.

QUESTION 21.
Which one of the following factors is responsible for excessive soil erosion in Chhotanagpur plat
eau?

a) Heavy rain throughout the year


b) Loose sandy soil
c) Deep ploughing by tractors
d) Large scale felling of trees
Correct Answer: D
Your Answer: Unanswered
Explanation

Solution (d)

The Chota Nagpur Plateau is a plateau in eastern India, which covers much
of Jharkhand state as well as adjacent parts of Odisha, West Bengal, Bihar and Chhattisgarh.
The Indo-Gangetic plain lies to the north and east of the plateau, and the basin of the Mahanadi
River lies to the south.

More than half of the natural forest on the plateau has been cleared for grazing land and the
scale of the mining operations on the plateau . The reckless destruction of forests in Chota
Nagpur Plateau has resulted in serious problem of soil erosion

IASbaba
Web: http://ilp.iasbaba.com/ Score:
Email: ilp@iasbaba.com 0.00 / 200
Page 19
Set 3 Block 1:
Exam Title :
Geography & ...
Email : misrapulkit@yahoo.in
Contact :

QUESTION 22.
Which of the following statements in regard to the peninsular rivers of the Indian subcontinent
is true?

a) These flow through deep gorges


b) These have perennial source of water supply
c) Most of these are dependent on rainfall
d) Meandering course is their chief characteristic
Correct Answer: C
Your Answer: Unanswered
Explanation

Solution (c)

The origins of peninsular rivers lie in plateaus and small hills. There is no snow to feed water,
and as such these rivers are seasonal and dry up in summers. These rivers do not have a high
erosion activity as they flow through gentle slopes. The flow of water in these rivers is also at a
slow pace, not allowing for meandering of rivers. However, these rivers still prove abundant
source of hydroelectricity.

QUESTION 23.
Consider the following statements regarding the Himalayas

1. The Himalayan rivers are older than the mountain ranges they cross
2. The Himalayan rivers cut their channels deeply and incisively

Which of the above statement(s) is/are correct?

a) 1 only
b) 2 only
c) 1 and 2
d) Neither 1 nor 2
Correct Answer: C
Your Answer: Unanswered
Explanation

Solution (c)

The Himalayan Rivers are great examples of antecedent origin . These rivers originated well
before the Himalayan region was uplifted. The rivers Indus, Brahmaputra, Sutlej, Kosi and
Subansiri originated on Tibetan side and now traverse the existing mountain ranges, cutting
deep gorges.

QUESTION 24.

IASbaba
Web: http://ilp.iasbaba.com/ Score:
Email: ilp@iasbaba.com 0.00 / 200
Page 20
Set 3 Block 1:
Exam Title :
Geography & ...
Email : misrapulkit@yahoo.in
Contact :

Which of the following have coral reefs?

1. Andaman and Nicobar Islands


2. Gulf of Kutch
3. Gulf of Mannar
4. Sunderbans

Select the correct answer using the codes given below

a) 1, 2 and 3
b) 2 and 4
c) 1 and 3
d) All of the above
Correct Answer: A
Your Answer: Unanswered
Explanation

Solution (a)

Coral reefs in India are one of the most ancient and dynamic ecosystems of India. The coral
reefs not only provide a sanctuary to a myriad of marine life but also play a key role in
protecting the coastline from erosion. India has about 7517 km of coastline including islands
but mainland coast is 6100 km.

• Andaman and Nicobar Islands Situated in the Bay of Bengal, exclusively fringing reefs
of about 500 islands, most of these islands have a healthy biodiversity.
• Gulf of Kutch Exclusively consists of fringing reefs. The reefs are relatively less
developed due to large range of temperature and high salinity. The harbours have less
biodiversity. The entire Gulf of Kutch is also known as a marine national park.
• Gulf of Mannar Fringing reefs with a chain of 21 islands from Rameswaram in the north
to Thootukudi (Tuticorin) in the south. This part of the gulf forms part of the Gulf of
Mannar biosphere reserve
• Lakshadweep Exclusively coral atolls with 36 islands, of which 10 are inhabited. The
islands range from less than 1 km (0.62 mi) to about 9 km (5.6 mi) in length, and do not
exceed 2 km (1.2 mi) in width.
• Others Tarkarli in Malwan , Maharashtra is a smaller reef. There are some coral reefs
around small inlets in the western part of the Gulf of Khambat. Angria Bank is a coral reef
off Vijaydurg in Maharashtra. There is a coral reef in Netrani Island in Karnataka.

QUESTION 25.
The most plausible explanation for the location of the Thar desert in Western India is

a) The moisture carried by the south-west monsoon is driven away by the dry upper air
current
b) The obstruction caused by the Aravalis to the rain- bearing wind that proceeds to the
Ganga valley
c) The evaporation of moisture by heat
d) The absence of mountains to the north of Rajasthan to cause Orographic rainfall in it
Correct Answer: D
Your Answer: Unanswered

IASbaba
Web: http://ilp.iasbaba.com/ Score:
Email: ilp@iasbaba.com 0.00 / 200
Page 21
Set 3 Block 1:
Exam Title :
Geography & ...
Email : misrapulkit@yahoo.in
Contact :
Explanation

Solution (d)

Thar Desert remains practically dry because there is no transverse mountain range to check the
Monsoon which directly advances towards the Kashmir Himalaya. Of course, there is a
mountain range in Rajasthan namely the Aravalli Hills but they stretch South-West to North-
East direction. Thus, the Aravalli Hills are roughly parallel to the Arabian Sea Monsoon.

QUESTION 26.
Consider the following definitions

1. Catchment area- River drains the water collected from specific area.
2. Drainage Basin-Boundary line separating one watershed from other.
3. Watershed-An area drained by a river and its tributaries.

Which of the given statements are INCORRECT?

a) 1 and 2
b) 2 and 3
c) 1 and 3
d) 1, 2 and 3
Correct Answer: B
Your Answer: Unanswered
Explanation

Solution (b)

Catchment area: It is a specific area from which a river drains the water collected .All the
surface water from rain, melting snow or ice converges to a single point at a lower elevation,
usually the exit of the basin, where the waters join another body of water, such as a river, lake,
reservoir, estuary, wetland, sea, or ocean.

IASbaba
Web: http://ilp.iasbaba.com/ Score:
Email: ilp@iasbaba.com 0.00 / 200
Page 22
Set 3 Block 1:
Exam Title :
Geography & ...
Email : misrapulkit@yahoo.in
Contact :

Drainage Basin: It is an area drained by a river and its tributaries. Just as a bathtub catches
all of the water that falls within its sides, a drainage basin sends all of the water falling within it
to a central river and out to an estuary or to the ocean. A drainage basin drains all of the land
around a major river.

IASbaba
Web: http://ilp.iasbaba.com/ Score:
Email: ilp@iasbaba.com 0.00 / 200
Page 23
Set 3 Block 1:
Exam Title :
Geography & ...
Email : misrapulkit@yahoo.in
Contact :

Watershed : It is a boundary line separating one drainage basin from the other one. It is an
area or ridge of land that separates waters flowing to different rivers, basins, or seas.

IASbaba
Web: http://ilp.iasbaba.com/ Score:
Email: ilp@iasbaba.com 0.00 / 200
Page 24
Set 3 Block 1:
Exam Title :
Geography & ...
Email : misrapulkit@yahoo.in
Contact :

Do you know?

• The catchments of large rivers are called river basins while those of small rivulets are rills
which are often referred as Watersheds
• The main difference between river basin and watersheds are , watersheds are small in
area while the basins cover larger areas

THINK!

• Water Divide
• Tributaries, Deltas, Riverine Islands, Estuaries

QUESTION 27.
Consider the following regarding Brahmaputra River

a) It has more silt in its Tibetan drainage basin.


b) It has less silt in its Indian drainage basin.
c) Both a and b are correct.
d) None of the above.

IASbaba
Web: http://ilp.iasbaba.com/ Score:
Email: ilp@iasbaba.com 0.00 / 200
Page 25
Set 3 Block 1:
Exam Title :
Geography & ...
Email : misrapulkit@yahoo.in
Contact :
Correct Answer: D
Your Answer: Unanswered
Explanation

Solution (d)

The Brahmaputra is one of the major rivers of Asia, a trans-boundary river which flows through
China, India, Bangladesh and Tibet

The Brahmaputra River which is known as Tsangpo in Tibet, receives very little volume of water
in Tibet, so it has less silt in Tibetan drainage basin.

Whereas when the same river enters into India, it passes through a region which receives heavy
rainfall (Assam and Arunachal Pradesh)and hence it has more silt in its Indian drainage basin

Do you know?

Brahmaputra is the tenth largest river in the world by discharge, and the fifteenth longest in
the world

IASbaba
Web: http://ilp.iasbaba.com/ Score:
Email: ilp@iasbaba.com 0.00 / 200
Page 26
Set 3 Block 1:
Exam Title :
Geography & ...
Email : misrapulkit@yahoo.in
Contact :

THINK!

• Ganga river
• Indus river
• States in which all the Himalayan river passes through

QUESTION 28.
Consider the following about east and west flowing rivers

1. East flowing rivers make deltas at their mouth whereas west flowing rivers enter the sea
through estuaries.
2. Tributaries of east flowing rivers are comparatively large compared to west flowing rivers.
3. East flowing rivers will not flow through very deep channels whereas west flowing rivers
flow through deep channels.

Select the correct statement/s

a) 1 and 2
b) 2 and 3
c) 1 and 3
d) 1,2 and 3
Correct Answer: D
Your Answer: Unanswered
Explanation

Solution (d)

All the statements are correct.

IASbaba
Web: http://ilp.iasbaba.com/ Score:
Email: ilp@iasbaba.com 0.00 / 200
Page 27
Set 3 Block 1:
Exam Title :
Geography & ...
Email : misrapulkit@yahoo.in
Contact :

IASbaba
Web: http://ilp.iasbaba.com/ Score:
Email: ilp@iasbaba.com 0.00 / 200
Page 28
Set 3 Block 1:
Exam Title :
Geography & ...
Email : misrapulkit@yahoo.in
Contact :

Do you know?

• Godavari is the largest peninsular river system in India.


• Kaveri River carries water throughout the year with comparatively less fluctuation than
the other peninsular rivers.

Think!

• Why are peninsular rivers non-perennial?

QUESTION 29.
Consider the following statements regarding river Kosi

1. It is also known as 'SAPTAKOSI'.


2. It brings huge quantity of sediments from its upper reaches and deposits it in the plains.
3. It has been notorious for frequently changing its course.
4. It flows through Tibet and India only.

Select the correct statement/s

IASbaba
Web: http://ilp.iasbaba.com/ Score:
Email: ilp@iasbaba.com 0.00 / 200
Page 29
Set 3 Block 1:
Exam Title :
Geography & ...
Email : misrapulkit@yahoo.in
Contact :

a) 1,2 and 4
b) 1,3 and 4
c) 1, 2 and 3
d) 1, 2, 3 and 4
Correct Answer: C
Your Answer: Unanswered
Explanation

Solution (c)

River Kosi is also known as 'SAPTAKOSI' in Nepal. It brings huge quantity of sediments from
its upper reaches and deposits it in the plains. The course gets blocked and consequently the
river changes its course. It has been notorious for frequently changing its course. It flows
through Tibet, Nepal and India

Do you know?

• River Kosi is also known as 'SORROW OF BIHAR' as the annual floods affect about
21,000 km2 of fertile agricultural lands thereby disturbing the rural economy.

THINK!

• Why does the Kosi River bring huge quantity of sediments from the upper reaches?
• What would be the remedial measures for controlling floods created by Kosi River?

IASbaba
Web: http://ilp.iasbaba.com/ Score:
Email: ilp@iasbaba.com 0.00 / 200
Page 30
Set 3 Block 1:
Exam Title :
Geography & ...
Email : misrapulkit@yahoo.in
Contact :
QUESTION 30.
Indo-Brahma river got dismembered into three drainage systems due to

a) Pleistocene upheaval in the western Himalayas.


b) Uplifting of the potwar plateau/Delhi ridge.
c) Both a and b are correct.
d) None of the above
Correct Answer: C
Your Answer: Unanswered
Explanation

Solution (c)

There is difference of opinion about the evolution of the Himalayan Rivers. However geologists
believe that a mighty river called Shiwalik or Indo-Brahma traversed the entire longitudinal
extent of the Himalaya from Assam to Punjab and onwards to Sindh and finally discharged into
the gulf of Sind near lower Punjab during the Miocene period some 5-24 million years ago.

The remarkable continuity of the Shiwalik and its lacustrine origin and alluvial deposits
consisting of sands, salts, clay, boulders and conglomerates support this view point.

It is opined that in due course of time Indo-Brahma river was dismembered into 3 main
drainage systems: Indus, Ganga and Brahmaputra.

This dismemberment was probably due to the Pleistocene ( from about 2 million to 10,000 years
ago) upheaval in the western Himalayas and uplifting of the potwar plateau (Delhi Ridge),
which acted as the water divide between the Indus and Ganga drainage systems

Do you know?

• The down thrusting of the Malda gap area between Rajmahal hills and the Meghalaya
plateau during the mid-Pleistocene period diverted both Ganga and Brahmaputra river
systems to flow towards the Bay of Bengal

THINK!

• Evolution of Peninsular drainage

QUESTION 31.
Himalayan tributaries like Shyok , Nubra , Gilgit are part of

a) Indus river system


b) Ganga river system
c) Brahmaputra river system
d) Nepal river system
Correct Answer: A
Your Answer: Unanswered
Explanation

Solution (a)

IASbaba
Web: http://ilp.iasbaba.com/ Score:
Email: ilp@iasbaba.com 0.00 / 200
Page 31
Set 3 Block 1:
Exam Title :
Geography & ...
Email : misrapulkit@yahoo.in
Contact :

The Indus receives a number of Himalayan tributaries such as the Shyok, the Gilgit, the Zaskar,
the Hunza, the Nubra, the Shigar, the Gasting and the Dras. It finally emerges out of the hills
near Attock where it receives the Kabul river on its right bank.

Do you know?

• The Indus flows in India only through the Leh district in Jammu and Kashmir

THINK!

• Tributaries of Ganga and Brahmaputra river systems

IASbaba
Web: http://ilp.iasbaba.com/ Score:
Email: ilp@iasbaba.com 0.00 / 200
Page 32
Set 3 Block 1:
Exam Title :
Geography & ...
Email : misrapulkit@yahoo.in
Contact :
QUESTION 32.
Consider the following differences related to Himalayan Rivers and Peninsular rivers

1. Himalayan rivers flow throughout the year whereas Peninsular Rivers does not.
2. Himalayan rivers are useful for navigation and irrigation whereas peninsular rivers are
neither navigable nor useful for irrigation.

Choose the correct option

a) 1 only
b) 2 only
c) Both are correct.
d) Both are wrong
Correct Answer: C
Your Answer: Unanswered
Explanation

Solution (c)

Both the options are correct.

IASbaba
Web: http://ilp.iasbaba.com/ Score:
Email: ilp@iasbaba.com 0.00 / 200
Page 33
Set 3 Block 1:
Exam Title :
Geography & ...
Email : misrapulkit@yahoo.in
Contact :

QUESTION 33.
The River which feeds the canal system of Bhakra Nangal Project is

a) Beas
b) Satluj
c) Ravi
d) Chenab
Correct Answer: B
Your Answer: Unanswered
Explanation

Solution (b)

IASbaba
Web: http://ilp.iasbaba.com/ Score:
Email: ilp@iasbaba.com 0.00 / 200
Page 34
Set 3 Block 1:
Exam Title :
Geography & ...
Email : misrapulkit@yahoo.in
Contact :

Satluj River is the answer.

Satluj originates in the Rakas lake near Mansarovar at an altitude of 4555m in Tibet where it is
known as 'Langchen Khambab'. It flows parallel to Indus for about 400km before entering India,
and comes out as gorge at Rupar. It passes through the Shipki La on the Himalayan ranges and
enters the Punjab plains. It is a very important tributary of Indus system as it feeds the canal
system of the Bhakra Nangal Project.

Do you know?

• The Sutlaj River is also known as 'Satadree'. It is the easternmost tributary of the Indus
River .
• The waters of the Sutlej are allocated to India under the Indus Waters Treaty between
India and Pakistan, and are mostly diverted to irrigation canals in India.

THINK!

• Indus Water Treaty


• Hydroelectric projects across Sutlej

IASbaba
Web: http://ilp.iasbaba.com/ Score:
Email: ilp@iasbaba.com 0.00 / 200
Page 35
Set 3 Block 1:
Exam Title :
Geography & ...
Email : misrapulkit@yahoo.in
Contact :
QUESTION 34.
Consider the following statements regarding Indus River

1. It is called as 'SINGI KHAMBAN' in Nepal.


2. It originates from a glacier near Bokhar Chu in the kailash mountain range of Tibet
region.
3. It enters into Pakistan near chillar in Baltistan region.
4. It cuts across the ladakh range forming a spectacular gorge near Gilgit in Jammu and
Kashmir.

Select the INCORRECT statement/s

a) 2 and 4
b) 1 and 2
c) 1 and 3
d) 1 and 4
Correct Answer: C
Your Answer: Unanswered
Explanation

Solution (c)

The Indus River (also called the Sindhu) is a major south-flowing river in South Asia . It is one of
the largest river basins of the world. The Indus is the westernmost of the Himalayan Rivers in
India. It originates from a glacier near Bokhar Chu in the kailash mountain range of Tibet
region at an altitude of 4164 m in the Kailash Mountain range. It is called as 'SINGI KHAMBAN'
or Lion's Mouth in Tibet.

IASbaba
Web: http://ilp.iasbaba.com/ Score:
Email: ilp@iasbaba.com 0.00 / 200
Page 36
Set 3 Block 1:
Exam Title :
Geography & ...
Email : misrapulkit@yahoo.in
Contact :

After flowing in the northwest direction between ladakh and Baltistan, it cuts across the Ladakh
range forming a spectacular gorge near Gilgit in Jammu and Kashmir. It enters into Pakistan
near Chillar in the Dardistan region

Do you know?

• It is the longest river and national river of Pakistan .

IASbaba
Web: http://ilp.iasbaba.com/ Score:
Email: ilp@iasbaba.com 0.00 / 200
Page 37
Set 3 Block 1:
Exam Title :
Geography & ...
Email : misrapulkit@yahoo.in
Contact :
• The ultimate source of the Indus is in Tibet ; the river begins at the confluence of the Seng
ge Zangbo and Gar Tsangpo rivers that drain the Nganglong Kangri and Gangdise Shan
(Gang Rinpoche, Mt. Kailas) mountain ranges.

THINK!

• Ganga river system


• Brahmaputra river system
• Laws and Current happenings related to Indus Water Treaty

QUESTION 35.
The River which rises from a spring at Verinag situated at the foot of Pir Panjal is

a) Satluj
b) Chenab
c) Ravi
d) Jhelum
Correct Answer: D
Your Answer: Unanswered
Explanation

Solution (d)

Jhelum is an important tributary of the Indus which rises from a spring at Verinag situated at
the foot of the Pir Panjal in the south eastern part of the valley of Kashmir.It flows through
Srinagar and the Wular lake before entering Pakistan through a deep narrow gorge. It joins the
Chenab river near Jhang in Pakistan.

Do you know?

• It is the westernmost of the five rivers of Punjab


• The Sanskrit name of this river is Vitasta.

QUESTION 36.
Consider the following statements with respect to Ganga river system

1. It rises in the Gangotri glacier near Gaumukh in the Uttarkashi district of Himachal
Pradesh where it is called as Bhagirathi.
2. It is the largest river system in India having a number of perennial and non-perennial
rivers.
3. Yamuna is the largest tributary of Ganga.

Select the correct statement/s

a) 1 and 2
b) 2 and 3
c) 1 and 3
d) 1,2 and 3

IASbaba
Web: http://ilp.iasbaba.com/ Score:
Email: ilp@iasbaba.com 0.00 / 200
Page 38
Set 3 Block 1:
Exam Title :
Geography & ...
Email : misrapulkit@yahoo.in
Contact :
Correct Answer: B
Your Answer: Unanswered
Explanation

Solution (b)

Ganga is the most important river of India w.r.t its basin and cultural significance. It rises in
the gangotri glacier near Gaumukh(3900m) in the Uttarkashi district of Uttara Khand where it
is called as Bhagirathi. Ganga is a trans-boundary river of Asia which flows through the nations
of India and Bangladesh .

The Ganges was ranked as the fifth most polluted river of the world in 2007. Pollution threatens
not only humans, but also more than 140 fish species, 90 amphibian species and the
endangered Ganges river dolphin .

The main stem of the Ganges begins at

· the confluence of the Bhagirathi and Alaknanda rivers in the town of Devprayag in the Garhwa
l division of the Indian state of Uttarakhand .

The Bhagirathi is considered to be the source in Hindu culture and mythology, although the
Alaknanda is longer, and, therefore, hydrologically the source stream. The Bhagirathi rises at
the foot of Gangotri Glacier , at Gomukh , at an elevation of 3,892 m (12,769 ft). The Ganges
Delta , formed mainly by the large, sediment-laden flows of the Ganges and Brahmaputra rivers,
is the world's largest delta, at about 59,000 km2

It is the largest river system in India having a number of perennial and non-perennial rivers.
Yamuna, is the largest tributary of Ganga has its source in Yamunotri glacier on the western
slopes of Banderpunch range.. The river has a length of 2525 km which is shared by
Uttarakhand(110km), Uttar Pradesh(1450km), Bihar(445km) and West Bengal(520km).The son
is a major right bank tributary

IASbaba
Web: http://ilp.iasbaba.com/ Score:
Email: ilp@iasbaba.com 0.00 / 200
Page 39
Set 3 Block 1:
Exam Title :
Geography & ...
Email : misrapulkit@yahoo.in
Contact :

Do you know?

• Ganges-Brahmaputra is the third largest river in the world by discharge followed by


Amazon(first) in South America and Congo(second) of Africa.
• After entering Bangladesh , the main branch of the Ganges is known as the Padma .
• It is worshipped as the goddess Ganga in Hinduism .

THINK!

• origin of tributaries of river Ganga


• Dams built across river Ganga

QUESTION 37.
River Yamuna, the western most and longest tributary of Ganga traverses through how many
states?

a) 4 states
b) 2 states
c) 3 states
d) 5 states
Correct Answer: A
Your Answer: Unanswered
Explanation

Solution (a)

• The Yamuna also known as the Jumna, (not to be mistaken with the Jamuna of
Bangladesh) is the longest and the second largest tributary river of the Ganges (Ganga) in
northern India . Originating from the Yamunotri Glacier at a height of 6,387 metres on the
south western slopes of Banderpooch peaks in the uppermost region of the Lower
Himalaya in Uttarakhand ,

IASbaba
Web: http://ilp.iasbaba.com/ Score:
Email: ilp@iasbaba.com 0.00 / 200
Page 40
Set 3 Block 1:
Exam Title :
Geography & ...
Email : misrapulkit@yahoo.in
Contact :

· It crosses 4 states, Uttarakhand, Himachal Pradesh, Haryana and Uttar Pradesh . Tons River ,
Yamuna's largest tributary, rises in the 20,720 ft (6,315 m) high Bandarpoonch mountain, and
has a large basin in Himachal Pradesh . It meets Yamuna below Kalsi near Dehradun ,
Uttarakhand.

IASbaba
Web: http://ilp.iasbaba.com/ Score:
Email: ilp@iasbaba.com 0.00 / 200
Page 41
Set 3 Block 1:
Exam Title :
Geography & ...
Email : misrapulkit@yahoo.in
Contact :

The water of Yamuna is of "reasonably good quality" through its length from Yamunotri in the
Himalayas to Wazirabad in Delhi.

Do you know?

In 1909 the waters of the Yamuna were distinguished as "clear blue", as compared to the silt-
laden yellow of the Ganges.

THINK!

• Triveni Sangam

IASbaba
Web: http://ilp.iasbaba.com/ Score:
Email: ilp@iasbaba.com 0.00 / 200
Page 42
Set 3 Block 1:
Exam Title :
Geography & ...
Email : misrapulkit@yahoo.in
Contact :
• Mythology of Yamuna

QUESTION 38.
Consider the following about river Chambal

1. It rises near Mhow in the malwa plateau of Uttar Pradesh.


2. It is famous for its badland topography called Chambal ravines.
3. Bansagar dam is one of the 4 major dams built across this river.

Select the incorrect statement/s

a) 1 and 2
b) 2 and 3
c) 1 and 3
d) 1,2 and 3
Correct Answer: C
Your Answer: Unanswered
Explanation

Solution (c)

The Chambal River is a tributary of the Yamuna River in central India , and thus forms part of
the greater Gangetic drainage system. The Chambal River is considered pollution free, and
hosts an amazing riverine faunal assemblage including 2 species of crocodilians – the mugger
and gharial , 8 species of freshwater turtles, smooth-coated otters, gangetic river dolphins,
skimmers, black-bellied terns, sarus cranes and black-necked storks, amongst others.

IASbaba
Web: http://ilp.iasbaba.com/ Score:
Email: ilp@iasbaba.com 0.00 / 200
Page 43
Set 3 Block 1:
Exam Title :
Geography & ...
Email : misrapulkit@yahoo.in
Contact :

IASbaba
Web: http://ilp.iasbaba.com/ Score:
Email: ilp@iasbaba.com 0.00 / 200
Page 44
Set 3 Block 1:
Exam Title :
Geography & ...
Email : misrapulkit@yahoo.in
Contact :

The chambal rises near Mhow in the malwa plateau of Madhya Pradesh and flows northwards
through a gorge upwards of Kota in rajasthan, where the Gandhisagar dam has been
constructed. The Chambal is famous for its badland topography called Chambal ravines.

The tributaries of the Chambal include Shipra , Choti Kalisindh, Sivanna, Retam, Ansar,
Kalisindh, Banas, Parbati, Seep, Kuwari, Kuno, Alnia, Mej, Chakan, Parwati, Chamla, Gambhir,
Lakhunder, Khan, Bangeri, Kedel and Teelar.

Do you know?

• The Gandhi Sagar dam is the first of the four dams built on the Chambal River.
• It is a legendary river and finds mention in ancient scriptures.

Think!

• What are the reasons for ravines formation in Chambal river?

QUESTION 39.
Consider the following tributaries and their origins of river Ganga

1. Ramganga - originates in the Nepal Himalayas between Dhaulagiri and Mount Everest.
2. Gandak -originates in the glaciers of Mapchachungo .
3. Son-originates in the Garhwal hills near Gairsain .
4. Ghaghara-originates in the Amarkantak plateau

Select the correct match

a) 1 and 3 only

IASbaba
Web: http://ilp.iasbaba.com/ Score:
Email: ilp@iasbaba.com 0.00 / 200
Page 45
Set 3 Block 1:
Exam Title :
Geography & ...
Email : misrapulkit@yahoo.in
Contact :

b) 2 and 4 only
c) All of the above
d) None of the above
Correct Answer: D
Your Answer: Unanswered
Explanation

Solution (d)

None of them are correctly matched

River Ganga has many tributaries like Ramganga, Gandak, Ghaghara, Kosi ,Damodar, Son etc

Ramganga is comparitively a small river rising in the Garhwal hills near Gairsain. It changes
its course to the south western direction after crossing the shiwalik and enters into the plains of
Uttar Pradesh near Najibabad. Finally it joins the Ganga near Kannauj.

Gandak comprises two streams anmely kaligandak and Trishulganga. It rises in the Nepal
Himalayas between the Dhaulagiri and Mount Everest and drains the central part of Nepal.

Son is a large soth bank tributary of the ganga, originating in the amarkantak plateau. After
forming a series of waterfalls at the edge of the plateau, it reaches Arrah, west of Patna to join
the Ganga.

Ghaghara originates in the glaciers of Mapchachungo. The river Sarda (Kali or Kali Ganga)
joins it in the plain before it finally meets the Ganga at Chhapra.

River Kosi is also known as 'SAPTAKOSI' in Nepal . It brings huge quantity of sediments from
its upper reaches and deposits it in the plains. The course gets blocked and consequently the

IASbaba
Web: http://ilp.iasbaba.com/ Score:
Email: ilp@iasbaba.com 0.00 / 200
Page 46
Set 3 Block 1:
Exam Title :
Geography & ...
Email : misrapulkit@yahoo.in
Contact :
river changes its course. It has been notorious for frequently changing its course. It flows
through Tibet, Nepal and India

Do you know?

• River Damodar was also known as 'SORROW OF BENGAL'. But presently this river is
tamed by Damodar valley corporation, a multipurpose project

THINK!

• Damodar Valley Corporation


• Other tributaries of Ganga
• Hills and Ghats across river Ganga

QUESTION 40.
The present drainage systems of peninsular India are caused due to:

1. Subsidence of the western flank of the peninsula leading to its submergence below the sea
during the early tertiary period.
2. Upheaval of the Himalayas when the northern flank of peninsular block was subjected to
subsidence and the consequent trough faulting.
3. Slight tilting of the peninsular block from North West to the south east direction gave
orientation to the entire drainage system towards Arabian Sea during the same period.

Choose the correct statement/s

a) 1 and 3
b) 1 and 2
c) 2 and 3
d) 1,2 and 3
Correct Answer: B
Your Answer: Unanswered
Explanation

Solution (b)

The present drainage systems of peninsular India are caused due to three major geological
events that had occurred in the distant past which are as follows

IASbaba
Web: http://ilp.iasbaba.com/ Score:
Email: ilp@iasbaba.com 0.00 / 200
Page 47
Set 3 Block 1:
Exam Title :
Geography & ...
Email : misrapulkit@yahoo.in
Contact :

1. Subsidence of the western flank of the peninsula leading to its submergence below the sea
during the early tertiary(66 million to 2.6 million years ago) period. Generally it has
disturbed the symmetrical plan of the river on either side of the original watershed.
2. Upheaval of the Himalayas when the northern flank of peninsular block was subjected to
subsidence and the consequent trough faulting. The Narmada and Tapi flow in trough
faults and fill the original cracks with their detritus materials. Hence, there is a lack of
alluvial and deltaic deposits in these rivers.
3. Slight tilting of the peninsular block from North West to the south east direction gave
orientation to the entire drainage system towards Bay of Bengal during the same period.

THINK!

• Evolution of Himalayan drainage.

QUESTION 41.
River Mahanadi flows through which of the below states?

a) Odisha, Jharkhand and Maharashtra.

IASbaba
Web: http://ilp.iasbaba.com/ Score:
Email: ilp@iasbaba.com 0.00 / 200
Page 48
Set 3 Block 1:
Exam Title :
Geography & ...
Email : misrapulkit@yahoo.in
Contact :

b) Jharkhand, Chhattisgarh and West Bengal.


c) Odisha, Chhattisgarh and Madhya Pradesh.
d) Madhya Pradesh, Jharkhand and West Bengal.
Correct Answer: C
Your Answer: Unanswered
Explanation

Solution (c)

The Mahanadi is a major river in East Central India .The Mahanadi rises near Sihawa in
Raipur district of chhattisgarh and runs through odisha to discharge its water ino the Bay of
Bengal. Some navigation is carried on in the lower course of this river. It flows through
MadhyaPradesh,Chattisgarh and Odisha .

Prior to the construction of Hirakud Dam, the Mahanadi was navigable from its mouth up to
Arrang, about a 190 kilometres (120 mi) from its source. However numerous barrages apart
from the Hirakud have put an end to that. Today, boats are restricted to the delta region and the
Hirakud reservoir.

Do you know?

• 53% of the Mahanadi drainage basin is in Madhya Pradesh and Chhattisgarh, while 47 %
lies in Odisha.

IASbaba
Web: http://ilp.iasbaba.com/ Score:
Email: ilp@iasbaba.com 0.00 / 200
Page 49
Set 3 Block 1:
Exam Title :
Geography & ...
Email : misrapulkit@yahoo.in
Contact :
• The Mahanadi was notorious for its devastating floods for much of recorded history. Thus
it was called 'the sorrow of Orissa'

THINK!

• Hirakud Dam
• Trade and Agriculture benefits.

QUESTION 42.
Which one of the following is the place of confluence of the Alaknanda and Bhagirathi Rivers?

a) Vishnu Prayag
b) Karan Prayag
c) Deva Prayag
d) Rudra Prayag
Correct Answer: C
Your Answer: Unanswered
Explanation

Solution (c)

Ganga rises in the gangotri glacier in Uttarakhand and it is known as Bhagirathi here. It cuts
through the cental and lesser himalayas in narrow gorges. Alaknanda has its source in the
Satopanth glacier above Badrinath. At Devprayag in Uttarkhand, the Bhagirathi meets the
Alaknanda after which the river is known as Ganga. The river gets the name "Ganges" beyond D
evprayag . The alaknanada consists of Dhauli and the vishnuganga which meets at joshimath/ V
ishnuprayag. The other tributaries of Alaknanda such as the Pindar joins it at Karna Prayag
while mandakini/kaliganga meets it at RudraPrayag.

IASbaba
Web: http://ilp.iasbaba.com/ Score:
Email: ilp@iasbaba.com 0.00 / 200
Page 50
Set 3 Block 1:
Exam Title :
Geography & ...
Email : misrapulkit@yahoo.in
Contact :

Do you know?

• " Devaprayāga" means "Godly Confluence" in Sanskrit. As per Hindu scriptures,


Devaprayaga is the sacred event of merging two heavenly rivers, Alakananda and Bhagira
thi , to form the holy Ganges.

THINK!

• Triveni Sangam
• Aquatic Species near the confluence
• Rudra Prayag

QUESTION 43.
Consider the following regarding river Godavari

1. It is the second largest peninsular river.


2. It is also called as ' Dakshin Ganga'.
3. It is navigable all through its stretch.
4. It rises in the Tryambakeshwar in the Nasik district of Maharashtra.

IASbaba
Web: http://ilp.iasbaba.com/ Score:
Email: ilp@iasbaba.com 0.00 / 200
Page 51
Set 3 Block 1:
Exam Title :
Geography & ...
Email : misrapulkit@yahoo.in
Contact :

Choose the correct statement/s

a) 1 and 3 only
b) 2 and 3 only
c) 1 and 4 only
d) 2 and 4 only
Correct Answer: D
Your Answer: Unanswered
Explanation

Solution (d)

The Godavari is the largest peninsular river system. It is the second longest river in India after
the Ganges . It is also called as Dakshin Ganga. It rises in the Tryambakeshwar in the Nasik
district of Maharashtra and discharges its water into the Bay of Bengal.

Its tributaries run through the states of Maharashtra, Madhya Pradesh, Chhattisgarh, Odisha,
Karnataka, Puducherry, Telangana and Andhra Pradesh. It is navigable only in its deltaic
stretch. The river after reaching Rajamundri splits into several branches forming a large delta.

IASbaba
Web: http://ilp.iasbaba.com/ Score:
Email: ilp@iasbaba.com 0.00 / 200
Page 52
Set 3 Block 1:
Exam Title :
Geography & ...
Email : misrapulkit@yahoo.in
Contact :

Do you know?

• The Godavari is subjected to heavy floods in its lower reaches.


• The Krishna Godavari Basin is one of the main nesting sites of the endangered Olive
Ridley sea turtle .

IASbaba
Web: http://ilp.iasbaba.com/ Score:
Email: ilp@iasbaba.com 0.00 / 200
Page 53
Set 3 Block 1:
Exam Title :
Geography & ...
Email : misrapulkit@yahoo.in
Contact :
• Duduma Waterfalls is 175 metres (574 ft) high and one of the highest waterfalls in
southern India. It is located on the Sileru river which forms boundary between Andhra
Pradesh and Odisha states.

THINK!

• Wild Life Sanctuaries, National Parks, Biosphere reserves across Godavari Stretch.
• Dams and Hydroelectricity generations across Godavari Stretch.
• Flora and Fauna

QUESTION 44.
Dhuandhar waterfalls is formed by river

a) Tapi
b) Mahanadi
c) Narmada
d) Krishna
Correct Answer: C
Your Answer: Unanswered
Explanation

Solution (c)

Dhuandhar waterfalls is in Jabalpur district of Madhya Pradesh. The Dhuandhar Falls are
located on Narmada River in Bhedaghat and are 30 meters high. The Narmada River, making its
way through the world-famous Marble Rocks , narrows down and then plunges in a waterfall
known as Dhuandhaar. The plunge, which creates a bouncing mass of mist, is so powerful that
its roar is heard from a far distance.

IASbaba
Web: http://ilp.iasbaba.com/ Score:
Email: ilp@iasbaba.com 0.00 / 200
Page 54
Set 3 Block 1:
Exam Title :
Geography & ...
Email : misrapulkit@yahoo.in
Contact :

Do you know?

• The word Dhuandhar is derived from two Hindi words - Dhuan (smoke) + Dhar (flow)
which means smoke like waterfall (this smoke is formed by water vapours or the smoke
cascade).
• The best time to visit Dhuandhar falls is during Sharad Purnima , when Narmada
Mahotsava is being celebrated. The white marbles appears spectacular when the moon
light falls on it and give it a silver like appearance.

THINK!

• Rift Valley
• West flowing rivers Narmada and Tapi

QUESTION 45.
Consider the following regarding small rivers of India

1. Bedthi and Dhadhar rivers flow towards east to join Bay of Bengal.
2. Subarnrekha and Baitarni rivers flow towards west to join Arabian Sea.

Select the correct options

IASbaba
Web: http://ilp.iasbaba.com/ Score:
Email: ilp@iasbaba.com 0.00 / 200
Page 55
Set 3 Block 1:
Exam Title :
Geography & ...
Email : misrapulkit@yahoo.in
Contact :

a) 1 only
b) 2 only
c) Both 1 and 2
d) None of the above
Correct Answer: D
Your Answer: Unanswered
Explanation

Solution (d)

Smaller rivers flowing towards west:

• The rivers flowing towards the Arabian Sea have short courses.

Some of the rivers flowing west are

· Shetruniji,Bhadra,Dhadhar,Sabarmati,Mahi,Vaitarna,Kalinadi,Bedti,Sharavati,Mandovi,Jauari,
Bharathapuzha,Periyar,Pamba etc.

Smaller rivers flowing towards east:

• There are large numbers of rivers flowing towards the east along with their tributaries.
• Some of them are Subarnrekha, Baitarni, Brahmani, Vamsadhara, Penner, Palar, Vaigai
etc.

THINK!

• Hydroelectricity plants , Bunds or Dams across small rivers

QUESTION 46.
Consider the following statements and select the correct one

a) The pattern of flow of water in a river channel over a period of 5 years is known as its
regime.
b) Ganga River has a monsoon regime during rainy season.
c) Both a and b are correct.
d) None of the above.
Correct Answer: B
Your Answer: Unanswered
Explanation

Solution (b)

The pattern of flow of water in a river channel over a period of one year is known as its regime.
River regime can describe one of two characteristics of a reach of an alluvial river :

• The variability in its discharge throughout the course of a year in response to


precipitation, temperature, evapotranspiration, and drainage basin characteristics.
• A series of characteristic power-law relationships between discharge and width, depth,
and slope.

IASbaba
Web: http://ilp.iasbaba.com/ Score:
Email: ilp@iasbaba.com 0.00 / 200
Page 56
Set 3 Block 1:
Exam Title :
Geography & ...
Email : misrapulkit@yahoo.in
Contact :

The north Indian River originating from the Himalayas are perennial as they are fed by glaciers
through snow melt and also receive rainfall water during rainy season. The rivers of south India
do not originate from glaciers and their flow pattern witnesses fluctuations. The flow increases
during monsoon rains. Thus, the regime of the rivers of south India is controlled by rainfall
which also varies from one part of the peninsular plateau to the other.

The discharge is the volume of water flowing in a river measured over time. It is measured
either in cusecs(cubic feet per second) or cumecs (cubic metres per second)

The Ganga has its minimum flow during the January-June period. The maximum flow is attained
either in August or September. The river thus has a monsoon regime during the rainy season.

THINK!

• River regimes of all major rivers in India.

QUESTION 47.
Some of the tributaries of river Krishna are

IASbaba
Web: http://ilp.iasbaba.com/ Score:
Email: ilp@iasbaba.com 0.00 / 200
Page 57
Set 3 Block 1:
Exam Title :
Geography & ...
Email : misrapulkit@yahoo.in
Contact :

a) Koyna , Tungbhadra , Bhima .


b) Penganga, Indravati , Pranhita .
c) Kabini , Bhavani , Amravati.
d) Subansiri , Sankosh , Kameng
Correct Answer: A
Your Answer: Unanswered
Explanation

Solution (a)

River Krishna is the second largest east flowing peninsular river which rises near
Mahabaleshwar in Sahyadri. Its total length is 1401km. The Koyna, Tungabhadra and the Bhima
are its major tributaries.

The delta of this river is one of the most fertile regions in India and was the home to ancient Sat
avahana and Ikshvaku Sun Dynasty kings. Vijayawada is the largest city on the River Krishna. It
causes heavy soil erosion during the monsoon floods. It flows fast and furious, often reaching
depths of over 75 feet (23 m). Ironically, there is a saying in Marathi: "Shant vaahate
Krishnamaai" which means "quiet flows Krishna". This term is used to describe that a person
should be as quiet as Krishna.

IASbaba
Web: http://ilp.iasbaba.com/ Score:
Email: ilp@iasbaba.com 0.00 / 200
Page 58
Set 3 Block 1:
Exam Title :
Geography & ...
Email : misrapulkit@yahoo.in
Contact :

There are four rivers that join the Krishna at a confluence known as Preeti Sangam, or "Lover's
Meeting Point" in the Satara District . These are Venna River , Urmodi River, Tarli River and Koy
na River .

Do you know?

• Of the total catchment area of the Krishna, 27 percent lies in Maharashtra, 44 percent in
Karnataka and 29 percent in erstwhile Andhra Pradesh.
• The Krishna River is the fourth-biggest river in terms of water inflows and river basin area
in India , after the Ganga , Godavari and Brahmaputra .
• The river is also called Krishnaveni.
• Godavari River is linked to the Krishna river by commissioning the Polavaram right bank
canal with the help of Pattiseema lift scheme in the year 2015 to augment water
availability to the Prakasam Barrage in Andhra Pradesh
• Mullayanagiri peak in Karnataka at an altitude of 1,930 m (6,330 ft) above msl , is the
highest point of the Krishna basin.

THINK!

• Wild Life Sanctuaries, National Parks, Biosphere reserves across Godavari Stretch.
• Dams and Hydroelectricity generations across Godavari Stretch.
• Flora and Fauna

QUESTION 48.
Which river was once known as 'Sorrow of Bengal'?

a) Kosi
b) Mahanadi
c) Damodar
d) Bhagirathi
Correct Answer: C
Your Answer: Unanswered
Explanation

Solution (c)

Damodar was once called 'Sorrow of Bengal'.

IASbaba
Web: http://ilp.iasbaba.com/ Score:
Email: ilp@iasbaba.com 0.00 / 200
Page 59
Set 3 Block 1:
Exam Title :
Geography & ...
Email : misrapulkit@yahoo.in
Contact :

Damodar River is a river flowing across the Indian states of West Bengal and Jharkhand .
Damodar occupies the eastern margins of the Chotanagar Plateau where it flows through a rift
valley and finally joins the Hugli river in West Bengal. Rich in mineral resources, the valley is
home to large-scale mining and industrial activity. Earlier known as the Sorrow of
Bengal because of its ravaging floods in the plains of West Bengal, the Damodar and its
tributaries have been somewhat tamed with the construction of several dams. It was the most
polluted river of India in 2003.

IASbaba
Web: http://ilp.iasbaba.com/ Score:
Email: ilp@iasbaba.com 0.00 / 200
Page 60
Set 3 Block 1:
Exam Title :
Geography & ...
Email : misrapulkit@yahoo.in
Contact :

It has a number of tributaries and subtributaries, such as Barakar , Konar , Bokaro , Haharo, Ja
munia , Ghari, Guaia, Khadia and Bhera. The Barakar is its main tributary.The Damodar and the
Barakar trifurcates the Chota Nagpur plateau. The rivers pass through hilly areas with great
force, sweeping away whatever lies in their path

IASbaba
Web: http://ilp.iasbaba.com/ Score:
Email: ilp@iasbaba.com 0.00 / 200
Page 61
Set 3 Block 1:
Exam Title :
Geography & ...
Email : misrapulkit@yahoo.in
Contact :

Do you know?

• The first dam was built across the Barakar River , a tributary of the Damodar river at Tilai
ya in 1953.
• Damodar Means "rope around the belly", derived from Sanskrit (dama) "rope" and (udara)
"belly". This is another name of the Hindu god Krishna, given to him because his foster-
mother tied him to a large urn.

THINK!

• Damodar Valley Corporation


• Dams and Hydroelectricity generations across Damodar Stretch.
• Steel Industres across Damodar valley.

QUESTION 49.
Consider the following statements

1. It is the largest tributary of Indus.


2. It is also known as Chandrabhaga .
3. It flows for more than 1000km before entering into Pakistan.

Choose the river which best suits the above statements

a) Jhelum
b) Chenab
c) Satluj
d) Beas
Correct Answer: B
Your Answer: Unanswered
Explanation

Solution (b)

River Chenab is the answer.

Chenab is the largest tributary of river Indus. This river is a major river of India and Pakistan .
It forms in the upper Himalayas in the Lahaul and Spiti district of Himachal Pradesh , India, and
flows through the Jammu region of Jammu and Kashmir into the plains of the Punjab , Pakistan.
The waters of the Chenab are allocated to Pakistan under the terms of the Indus Waters Treaty I
t is formed by two streams chandra and bhaga, which join at Tandi near Keylong in Himachal
Pradesh. Hence it is known as Chandrabhaga. The river flows for 1180km before enering into
Pakistan.

IASbaba
Web: http://ilp.iasbaba.com/ Score:
Email: ilp@iasbaba.com 0.00 / 200
Page 62
Set 3 Block 1:
Exam Title :
Geography & ...
Email : misrapulkit@yahoo.in
Contact :

IASbaba
Web: http://ilp.iasbaba.com/ Score:
Email: ilp@iasbaba.com 0.00 / 200
Page 63
Set 3 Block 1:
Exam Title :
Geography & ...
Email : misrapulkit@yahoo.in
Contact :

THINK!

• Dams and Hydroelectricity generations across Chenab Stretch.


• Laws and Current scenario related to Indus Water Treaty.
• Wild life sanctuaries, National parks, Biosphere reserves

IASbaba
Web: http://ilp.iasbaba.com/ Score:
Email: ilp@iasbaba.com 0.00 / 200
Page 64
Set 3 Block 1:
Exam Title :
Geography & ...
Email : misrapulkit@yahoo.in
Contact :
QUESTION 50.
Which among the following acts as a water divide between the major peninsular rivers of India?

a) Vindhya range
b) Satpura range
c) Balaghat range
d) Western Ghats
Correct Answer: D
Your Answer: Unanswered
Explanation

Solution (d)

Western Ghats is the answer.

Peninsular drainage system is older than the Himalayan one. This is evident from the broad,
largely-graded shallow valleys and the maturity of the rivers. The Western Ghats running close
to western coast acts as the water divide between the major peninsular rivers, discharging
their water in the Bay of Bengal and as small rivulets joining the Arabian Sea. Except Narmada
and Tapi most of the major peninsular rivers flow from west to east. The chambal, the Sind, the
Betwa, the Ken, the Son which originates in the northern part of peninsular belong to the ganga
river system. Peninsular rivers are characterised by fixed course, absence of meanders and non-
perennial flow of water but Narmada and Tapi which flows towards Arabian sea due to rift
valley are however exceptions.

Ambala is located on the water divide between th Indus and the Ganga river systems in the
north India.

IASbaba
Web: http://ilp.iasbaba.com/ Score:
Email: ilp@iasbaba.com 0.00 / 200
Page 65
Set 3 Block 1:
Exam Title :
Geography & ...
Email : misrapulkit@yahoo.in
Contact :

IASbaba
Web: http://ilp.iasbaba.com/ Score:
Email: ilp@iasbaba.com 0.00 / 200
Page 66
Set 3 Block 1:
Exam Title :
Geography & ...
Email : misrapulkit@yahoo.in
Contact :

THINK!

• Other water divides of India


• Flora and Fauna of Water Divide

QUESTION 51.
We feel a powdery material when we catch a butterfly by its wings. What is it?

a) Setae, a bunch of tiny scales, modified sensory structures


b) Saliva of the butterfly retained from its cocoon
c) Pheromones to attract the opposite sex
d) None of the above
Correct Answer: A
Your Answer: Unanswered
Explanation

Solution (a)

Both butterflies and moths belong to the order Lepidoptera meaning scaly wings. As the order
name Lepidoptera implies, the powdery material found on butterfly wings that sticks to our
fingers is a bunch of tiny scales, modified sensory structures called setae. Wings are made of

IASbaba
Web: http://ilp.iasbaba.com/ Score:
Email: ilp@iasbaba.com 0.00 / 200
Page 67
Set 3 Block 1:
Exam Title :
Geography & ...
Email : misrapulkit@yahoo.in
Contact :
two thin membranes covered with unicellular scales arranged loosely in rows. Thin layers of
chitin, a hardened protein make the scales. These scales strengthen and stabilize the wings.
The scales are of two kinds, one filled with colouring materials and the other minutely grooved
and surfaced.

Each scale is plugged into a socket on the wing with grids of high and low ribs and cells. The
cells hold the natural pigment molecules such as uric acid (white), carotenoid pigments (yellow,
orange), quercetin (red, purple) and melanin (brown). Each pigmented scale produces only one
type of pigment. Along with the pigments, the arrangement and spacing of the ribs and cells
also contribute to the colour. However, the vivid iridescent colours are the results of reflection
and refraction of light by the different size, pattern and spacing of the ribs on the scales.

Besides regulating temperature by either absorbing or reflecting sunlight, the loose


attachments and slippery nature of scales prevent butterflies from predators. Some species
have androconial scales helping in the dispersal of pheromones to attract the opposite sex.

QUESTION 52.
The report ‘Trouble in the Making? The Future of Manufacturing-Led Development’ is released
by

a) World Bank
b) World Trade Organisation
c) World Economic Forum
d) International Monetary Fund
Correct Answer: A
Your Answer: Unanswered
Explanation

Solution (a)

A new report from the World Bank Group’s Trade & Competitiveness Global Practice, Trouble in
the Making? The Future of Manufacturing-Led Development, explains that the criteria for
becoming a desirable manufacturing location are changing. Companies once influenced by the
prospect of inexpensive labour costs are beginning to favour locations that can better take
advantage of new technologies.

IASbaba
Web: http://ilp.iasbaba.com/ Score:
Email: ilp@iasbaba.com 0.00 / 200
Page 68
Set 3 Block 1:
Exam Title :
Geography & ...
Email : misrapulkit@yahoo.in
Contact :

Source: http://www.livemint.com/Opinion/CMG633rqCOgrczIAx75ftJ/To-tackle-the-
Indian-economys-woes-create-more-jobs.html

QUESTION 53.
Which one of the following statements appropriately describes the “fiscal stimulus”?

a) It is a massive investment by the Government in manufacturing sector to ensure the supply


of goods to meet the demand surge caused by rapid economic growth
b) It is Government’s intensive action on financial institutions to ensure disbursement of loans
to agriculture and allied sectors to promote greater food production and contain food
inflation
c) It is an extreme affirmative action by the Government to pursue its policy of financial
inclusion
d) It is an intense affirmative action of the Government to boost economic activity in the
country
Correct Answer: D
Your Answer: Unanswered
Explanation

Solution (d)

Fiscal stimulus refers to increasing government consumption or transfers or lowering taxes.


Effectively this means increasing the rate of growth of public debt, except that particularly
Keynesians often assume that the stimulus will cause sufficient economic growth to fill that gap
partially or completely.

Think

IASbaba
Web: http://ilp.iasbaba.com/ Score:
Email: ilp@iasbaba.com 0.00 / 200
Page 69
Set 3 Block 1:
Exam Title :
Geography & ...
Email : misrapulkit@yahoo.in
Contact :

· Fiscal Deficit

Source: https://thewire.in/185107/yashwant-sinha-is-a-reminder-of-the-importance-of-
keynes-for-the-indian-economy/

QUESTION 54.
The ‘Horn of Africa’ is bordered with which of the following water bodies?

1. Red Sea

2. Gulf of Guinea

3. Gulf of Aden

Select the correct code:

a) 1 and 2
b) 2 and 3
c) 1 and 3
d) All of the above
Correct Answer: C
Your Answer: Unanswered
Explanation

Solution (c)

IASbaba
Web: http://ilp.iasbaba.com/ Score:
Email: ilp@iasbaba.com 0.00 / 200
Page 70
Set 3 Block 1:
Exam Title :
Geography & ...
Email : misrapulkit@yahoo.in
Contact :

QUESTION 55.
Salween River flows through

a) China
b) Myanmar

IASbaba
Web: http://ilp.iasbaba.com/ Score:
Email: ilp@iasbaba.com 0.00 / 200
Page 71
Set 3 Block 1:
Exam Title :
Geography & ...
Email : misrapulkit@yahoo.in
Contact :

c) Both (a) and (b)


d) Neither (a) nor (b)
Correct Answer: C
Your Answer: Unanswered
Explanation

Solution (c)

The Salween River, known as the Nu in China and the Thanlwin in Burma, stretches over 2,800
kilometers from its source to the Andaman Sea. Known as the “Grand Canyon of the East,” the
river cuts deep into the earth to create spectacular views. Impressed by the landscape and the
remarkable biological diversity of the area in China through which the Salween flows, the
UNESCO World Heritage Committee proclaimed the region a World Heritage Site in 2003. Only
two months after receiving the World Heritage designation, the Yunnan provincial government
announced its intention to build a 13-dam cascade on China’s portion of the river.

IASbaba
Web: http://ilp.iasbaba.com/ Score:
Email: ilp@iasbaba.com 0.00 / 200
Page 72
Set 3 Block 1:
Exam Title :
Geography & ...
Email : misrapulkit@yahoo.in
Contact :

IASbaba
Web: http://ilp.iasbaba.com/ Score:
Email: ilp@iasbaba.com 0.00 / 200
Page 73
Set 3 Block 1:
Exam Title :
Geography & ...
Email : misrapulkit@yahoo.in
Contact :

QUESTION 56.
The MADAD Portal is associated with which of the Ministries?

a) Ministry of External Affairs


b) Ministry of Commerce and Industry
c) Ministry of Corporate Affairs
d) Ministry of Home Affairs
Correct Answer: A
Your Answer: Unanswered
Explanation

Solution (a)

MADAD is described as a "Consular Grievances Monitoring System"

MADAD portal seeks to significantly reform the linear process adopted to post grievances from
the originator of the grievance to the concerned Embassy or Consulate of India abroad and cut
down the time required for grievances to be sent from Delhi to our Missions abroad and to get
them to take necessary action.

QUESTION 57.
The management of Haj Pilgrimage comes under

a) Ministry of Minority Affairs


b) Ministry of Culture
c) Ministry of External Affairs
d) Ministry of Home Affairs
Correct Answer: A
Your Answer: Unanswered
Explanation

Solution (a)

The work related to management of Haj pilgrimage has been transferred from Ministry of
External Affairs to Ministry of Minority

Read More: http://www.thehindu.com/news/national/haj-pilgrimage-comes-under-


control-of-minority-affairs-ministry/article9193652.ece

Source: http://economictimes.indiatimes.com/news/politics-and-nation/haj-subsidy-to-
be-abolished-gradually-mukhtar-abbas-naqvi/articleshow/60987392.cms

QUESTION 58.
Which of the following statements is/are correct?

IASbaba
Web: http://ilp.iasbaba.com/ Score:
Email: ilp@iasbaba.com 0.00 / 200
Page 74
Set 3 Block 1:
Exam Title :
Geography & ...
Email : misrapulkit@yahoo.in
Contact :

1. Article 340 in The Constitution of India deals with Appointment of a Commission to


investigate the conditions of backward classes

2. The President of India has appointed a Commission to examine the sub-categorisation of


Other Backward Classes

Select the correct statements

a) 1 Only
b) 2 Only
c) Both 1 and 2
d) Neither 1 nor 2
Correct Answer: C
Your Answer: Unanswered
Explanation

Solution (c)

The President appointed a Commission to examine the sub-categorisation of Other Backward


Classes.

Sub categorization of the OBCs will ensure that the more backward among the OBC
communities can also access the benefits of reservation for educational institutions and
government jobs.

Article 340 - Appointment of a Commission to investigate the conditions of backward


classes

· The President may by order appoint a Commission consisting of such persons as he thinks fit
to investigate the conditions of socially and educationally backward classes within the territory
of India and the difficulties under which they labour and to make recommendations as to the
steps that should be taken by the Union or any State to remove such difficulties and to improve
their condition and as to the grants that should be made for the purpose by the Union or any
State the conditions subject to which such grants should be made, and the order appointing
such Commission shall define the procedure to be followed by the Commission

· A Commission so appointed shall investigate the matters referred to them and present to the
President a report setting out the facts as found by them and making such recommendations as
they think proper

· The President shall cause a copy of the report so presented together with a memorandum
explaining the action taken thereon to be laid before each House of Parliament.

Source: https://timesofindia.indiatimes.com/india/president-appoints-commission-to-
examine-sub-categorization-of-obcs/articleshow/60914780.cms

QUESTION 59.
Consider the following statements

1. El Nino Modoki is characterized by strong warming in the central tropical Pacific

IASbaba
Web: http://ilp.iasbaba.com/ Score:
Email: ilp@iasbaba.com 0.00 / 200
Page 75
Set 3 Block 1:
Exam Title :
Geography & ...
Email : misrapulkit@yahoo.in
Contact :

2. EL Nino and La Nina are the atmospheric changes associated with the warming and
cooling of the Pacific Ocean

3. El Nino is associated with the droughts in India, while La Nina is associated with excess
rainfall in India

Select the correct statements

a) 1 and 2
b) 2 and 3
c) 1 and 3
d) All of the above
Correct Answer: D
Your Answer: Unanswered
Explanation

Solution (d)

El Nino Modoki is a coupled ocean-atmosphere phenomenon in the tropical Pacific. It is


different from another coupled phenomenon in the tropical Pacific namely, El Nino.

Conventional El Nino is characterized by strong anomalous warming in the eastern equatorial


Pacific.

Whereas, El Nino Modoki is associated with strong anomalous warming in the central tropical
Pacific and cooling in the eastern and western tropical Pacific.

Associated with this distinct warming and cooling patterns the teleconnections are very
different from teleconnection patterns of the conventional El Nino.

The El Nino Modoki phenomenon is characterized by the anomalously warm central equatorial
Pacific flanked by anomalously cool regions in both west and east.

Such zonal SST gradients result in anomalous two-cell Walker Circulation over the tropical
Pacific, with a wet region in the central Pacific.

IASbaba
Web: http://ilp.iasbaba.com/ Score:
Email: ilp@iasbaba.com 0.00 / 200
Page 76
Set 3 Block 1:
Exam Title :
Geography & ...
Email : misrapulkit@yahoo.in
Contact :

IASbaba
Web: http://ilp.iasbaba.com/ Score:
Email: ilp@iasbaba.com 0.00 / 200
Page 77
Set 3 Block 1:
Exam Title :
Geography & ...
Email : misrapulkit@yahoo.in
Contact :

Source: http://www.business-standard.com/article/economy-policy/north-east-monsoon-
to-be-normal-on-neutral-el-nino-imd-117100301102_1.html

QUESTION 60.
‘India Energy Outlook’ is released by

a) The Energy and Resources Institute (TERI)


b) International Energy Agency (IEA)
c) Exxon Mobil
d) None of the above
Correct Answer: B
Your Answer: Unanswered
Explanation

Solution (b)

The International Energy Agency (IEA), an autonomous agency, was established in November
1974.

Its primary mandate was – and is – two-fold: to promote energy security amongst its member
countries through collective response to physical disruptions in oil supply, and provide
authoritative research and analysis on ways to ensure reliable, affordable and clean energy for
its 29 member countries and beyond.

The IEA carries out a comprehensive programme of energy co-operation among its member
countries, each of which is obliged to hold oil stocks equivalent to 90 days of its net imports.

The Agency’s aims include the following objectives:

IASbaba
Web: http://ilp.iasbaba.com/ Score:
Email: ilp@iasbaba.com 0.00 / 200
Page 78
Set 3 Block 1:
Exam Title :
Geography & ...
Email : misrapulkit@yahoo.in
Contact :

· Secure member countries’ access to reliable and ample supplies of all forms of energy; in
particular, through maintaining effective emergency response capabilities in case of oil supply
disruptions.

· Promote sustainable energy policies that spur economic growth and environmental protection
in a global context – particularly in terms of reducing greenhouse-gas emissions that contribute
to climate change.

· Improve transparency of international markets through collection and analysis of energy data.

· Support global collaboration on energy technology to secure future energy supplies and
mitigate their environmental impact, including through improved energy efficiency and
development and deployment of low-carbon technologies.

· Find solutions to global energy challenges through engagement and dialogue with non-
member countries, industry, international organisations and other stakeholders.

Source: http://www.thehindu.com/todays-paper/tp-business/coal-is-still-the-secret-of-
our-energy/article19782409.ece

QUESTION 61.
Consider the following statements about Gulf of Aden

1. It is located between Yemen and Somalia

2. Bab-el-Mandeb connects the Red Sea to the Gulf of Aden

Select the correct statements

a) 1 Only
b) 2 Only
c) Both 1 and 2
d) Neither 1 nor 2
Correct Answer: C
Your Answer: Unanswered
Explanation

Solution (c)

The Bab-el-Mandeb or Mandeb Strait is a strait located between Yemen on the Arabian
Peninsula, and Djibouti and Eritrea in the Horn of Africa. It connects the Red Sea to the Gulf of
Aden.

Gulf of Aden is a gulf located in the Arabian Sea between Yemen, on the south coast of the
Arabian Peninsula, and Somalia in the Horn of Africa.

IASbaba
Web: http://ilp.iasbaba.com/ Score:
Email: ilp@iasbaba.com 0.00 / 200
Page 79
Set 3 Block 1:
Exam Title :
Geography & ...
Email : misrapulkit@yahoo.in
Contact :

Source: http://www.thehindu.com/news/national/indian-navy-thwarts-piracy-attempt-in-
gulf-of-aden/article19809790.ece

QUESTION 62.
Consider the following statements about Kaziranga National Park

1. It is part of the middle Brahmaputra alluvial flood plains

IASbaba
Web: http://ilp.iasbaba.com/ Score:
Email: ilp@iasbaba.com 0.00 / 200
Page 80
Set 3 Block 1:
Exam Title :
Geography & ...
Email : misrapulkit@yahoo.in
Contact :

2. Wild Water Buffalo is only found in Kaziranga National Park

3. It was inscribed for being the world’s major stronghold of the Indian one-horned rhino,
having the single largest population of this species

Select the correct statements

a) 1 and 2
b) 2 and 3
c) 1 and 3
d) All of the above
Correct Answer: C
Your Answer: Unanswered
Explanation

Solution (c)

Kaziranga National Park represents one of the last unmodified natural areas in the north-
eastern region of India. Covering 42,996 ha, and located in the State of Assam it is the single
largest undisturbed and representative area in the Brahmaputra Valley floodplain. The
fluctuations of the Brahmaputra River result in spectacular examples of riverine and fluvial
processes in this vast area of wet alluvial tall grassland interspersed with numerous broad
shallow pools fringed with reeds and patches of deciduous to semi-evergreen woodlands.
Kaziranga is regarded as one of the finest wildlife refuges in the world. The park’s contribution
in saving the Indian one-horned rhinoceros from the brink of extinction at the turn of the 20th
century to harbouring the single largest population of this species is a spectacular conservation
achievement. The property also harbours significant populations of other threatened species
including tigers, elephants, wild water buffalo and bears as well as aquatic species including
the Ganges River dolphin. It is an important area for migratory birds.

River fluctuations by the Brahmaputra river system result in spectacular examples of riverine
and fluvial processes. River bank erosion, sedimentation and formation of new lands as well as
new water-bodies, plus succession between grasslands and woodlands represents outstanding
examples of significant and ongoing, dynamic ecological and biological processes. Wet alluvial
grasslands occupy nearly two-thirds of the park area and are maintained by annual flooding and
burning. These natural processes create complexes of habitats which are also responsible for a
diverse range of predator/prey relationships.

Kaziranga was inscribed for being the world’s major stronghold of the Indian one-horned rhino,
having the single largest population of this species, currently estimated at over 2,000 animals.
The property also provides habitat for a number of globally threatened species including tiger,
Asian elephant, wild water buffalo, gaur, eastern swamp deer, Sambar deer, hog deer, capped
langur, hoolock gibbon and sloth bear. The park has recorded one of the highest density of tiger
in the country and has been declared a Tiger Reserve since 2007. The park’s location at the
junction of the Australasia and Indo-Asian flyway means that the park’s wetlands play a crucial
role for the conservation of globally threatened migratory bird species. The Endangered Ganges
dolphin is also found in some of the closed oxbow lakes.

In India, Wild Water Buffalo is now largely restricted to Assam, Arunachal Pradesh, and Madhya
Pradesh. In Assam, the species is found in and around Manas Sanctuary, Laokhowa Sanctuary,
Kaziranga National Park, and Dibru Sanctuary. In Arunachal Pradesh, the species occurs in and
around Namdapha Sanctuary. There are two populations in Madhya Pradesh (Bastar district),
one in Indravati National Park, and another in Udanti Sanctuary, which might extend into
adjacent parts of Orissa.

IASbaba
Web: http://ilp.iasbaba.com/ Score:
Email: ilp@iasbaba.com 0.00 / 200
Page 81
Set 3 Block 1:
Exam Title :
Geography & ...
Email : misrapulkit@yahoo.in
Contact :

Source: http://www.thehindu.com/news/national/other-states/when-is-the-safari-trail-
at-kaziranga-national-park-opening/article19778686.ece

QUESTION 63.
Consider the following statements about National Crime Records Bureau (NCRB)

1. It is under the Ministry of Statistics and Programme Implementation

2. It publishes an annual report called Crime in India that records crime on the basis of the
FIRs registered in the police stations across the country

Select the correct statements

a) 1 Only
b) 2 Only
c) Both 1 and 2
d) Neither 1 nor 2
Correct Answer: B
Your Answer: Unanswered
Explanation

Solution (b)

The National Crime Records Bureau (NCRB) under the Ministry of Home Affairs is the nodal
agency for collection and dissemination of information related to crime in India. The NCRB
publishes an annual report called Crime in India, that records crime on the basis of the FIRs
registered in the police stations across the country. It is the only official source of crime data in
India, and it records among other things crime committed state-wise and offence-wise (e.g.,
murder, rape, cheating, theft).

An expert committee under the Ministry of Statistics and Programme Implementation has noted
that there is significant under-reporting of crimes under the NCRB for various reasons. For
example, there could be suppression of data and low registration of crimes because the police
know that their work is judged on the basis of this information. Also, sometimes victims of
crime may decide against reporting the incident with the police because they are afraid to
approach the police, or think the crime is not serious enough, etc. Also, note that the NCRB
follows the ‘principal offence rule’ for counting crime. This means that if many offences are
covered in a single registered criminal case, the NCRB will only count the most heinous of the
offences. For instance, a case of murder and rape, will only be counted as a case of murder (i.e.
principal offence) by the NCRB.

Source: http://economictimes.indiatimes.com/news/politics-and-nation/india-is-the-
safest-insists-tourism-minister-citing-vegas/articleshow/60942915.cms

QUESTION 64.
Consider the following statements about Nansen Refugee Award

1. It recognizes extraordinary humanitarian work on behalf of refugees, displaced, or stateless


peoples

IASbaba
Web: http://ilp.iasbaba.com/ Score:
Email: ilp@iasbaba.com 0.00 / 200
Page 82
Set 3 Block 1:
Exam Title :
Geography & ...
Email : misrapulkit@yahoo.in
Contact :

2. It is awarded by the United Nations High Commissioner for Refugees (UNHCR)

3. It is named after the Norwegian polar explorer and humanitarian who went on to serve in the
1920s as the first High Commissioner for Refugees for the League of Nations

Select the correct statements

a) 1 and 2
b) 2 and 3
c) 1 and 3
d) All of the above
Correct Answer: D
Your Answer: Unanswered
Explanation

Solution (d)

Few humanitarian honours possess the rich legacy of UNHCR’s Nansen Refugee Award.
Established in 1954, the award recognizes extraordinary humanitarian work on behalf of
refugees, displaced, or stateless peoples. The award is named after Fridtjof Nansen, the
Norwegian polar explorer and humanitarian who went on to serve in the 1920s as the first High
Commissioner for Refugees for the League of Nations. Nansen was awarded the 1922 Nobel
Peace Prize in recognition of his courageous and unfaltering work on behalf of World War I
refugees. The Nansen Refugee Award, through its laureates, aims to showcase his values of
perseverance and conviction in the face of adversity. The 2017 winner is a remarkable example
of these attributes.

The United Nations High Commissioner for Refugees (UNHCR), was established on December
14, 1950 by the United Nations General Assembly. UNHCR safeguards the rights and well-being
of refugees and stateless people. In more than six decades, the agency has helped tens of
millions of people restart their lives. UNHCR is on the front lines of the world’s major
humanitarian crises, including Syria, Iraq, Central African Republic, Afghanistan, South Sudan,
Democratic Republic of the Congo, and countless other emergencies.

News

· The UN Refugee Agency, UNHCR honoured Zannah Mustapha with the 2017 Nansen Refugee
Award at a ceremony in Geneva for founding a school in Maiduguri, Nigeria, the epicentre of
the Boko Haram insurgency.

· The school has stayed open throughout the conflict with Boko Haram, which has seen some
20,000 killed across the Lake Chad region, and millions more displaced.

· The school provides a free education, as well as free meals, uniforms and health care, to
children affected and displaced by violence. Those orphaned by the conflict on both sides are
welcomed into Mustapha’s classrooms as a sign of the reconciliation he hopes to achieve in the
region.

Source: https://scroll.in/latest/853536/google-celebrates-norwegian-explorer-and-
refugee-advocate-fridtjof-nansen-with-doodle

QUESTION 65.

IASbaba
Web: http://ilp.iasbaba.com/ Score:
Email: ilp@iasbaba.com 0.00 / 200
Page 83
Set 3 Block 1:
Exam Title :
Geography & ...
Email : misrapulkit@yahoo.in
Contact :

Rajiv Kumar Committee is associated with

a) Water management in north-east


b) Inter linking of rivers
c) Foreign Direct Investment
d) Urban Flooding
Correct Answer: A
Your Answer: Unanswered
Explanation

Solution (a)

The government has set up a high-level committee to evolve a strategy for management of
NER’s water resources.

The committee will facilitate optimising benefits of appropriate water management in the form
of hydro-electric power, agriculture, bio-diversity conservation, reduced flood damage erosion,
inland water transport, forestry, fishery and eco-tourism.

With the ministry of development of north-eastern region (DoNER) serving as the coordinating
point, the committee will submit its report as well as a plan of action by June next year. This will
involve dovetailing central and state government schemes across departments and autonomous
bodies.

The committee’s formation comes against the backdrop of floods that have brought life to a
standstill in Assam, Arunachal Pradesh and Manipur.

The committee will be headed by Rajiv Kumar, vice-chairman of NITI Aayog, the federal policy
think tank, and will include secretaries from the ministries of DoNER, power, water resources,
river development and Ganga rejuvenation, National Disaster Management Authority,
departments of border management and space and the chief secretaries of all the eight states of
the region.

Source: http://www.livemint.com/Politics/SNxWae2Uuiy91Cm32wYmCI/Govt-sets-up-
committee-for-water-management-in-northeast.html

QUESTION 66.
The SECURE Himalaya Project is launched by

a) Ministry of Environment, Forests and Climate Change ( MoEFCC )


b) International Union for Conservation of Nature
c) Both (a) and (b)
d) Neither (a) nor (b)
Correct Answer: A
Your Answer: Unanswered
Explanation

Solution (a)

IASbaba
Web: http://ilp.iasbaba.com/ Score:
Email: ilp@iasbaba.com 0.00 / 200
Page 84
Set 3 Block 1:
Exam Title :
Geography & ...
Email : misrapulkit@yahoo.in
Contact :

Project aims to ensure conservation of locally and globally significant biodiversity, land and
forest resources in high Himalayan ecosystem spread over four states viz. Himachal Pradesh,
Jammu and Kashmir, Uttarakhand and Sikkim.

It was launched by Union Ministry of Environment, Forests and Climate Change (MoEFCC) in
association with the United Nations Development Programme (UNDP).

The SECURE project aims at securing livelihoods, conservation, sustainable use and restoration
of high range Himalayan ecosystems.

The key focus areas of the project is protection of snow leopard and other endangered species
and their habitats and also securing livelihoods of people in region and enhancing enforcement
to reduce wildlife crime.

Source: https://timesofindia.indiatimes.com/india/centre-launches-six-year-secure-
himalaya-project/articleshow/60914913.cms

QUESTION 67.
‘Agenda for Action 2020’ is concerned with India and

a) African Union
b) BIMSTEC
c) G-20
d) European Union
Correct Answer: D
Your Answer: Unanswered
Explanation

Solution (d)

India-EU Agenda for Action 2020 – the roadmap for bilateral cooperation endorsed during the
13th India-EU Summit.

EU-India Agenda for Action-2020 - http://www.mea.gov.in/Images/attach/


EU_India_Agenda_for_Action_post_VC.pdf

Source: http://pib.nic.in/newsite/PrintRelease.aspx?relid=171462

QUESTION 68.
PMGDISHA is associated with

a) Rural Electrification
b) Digital literacy
c) Rural Employment Guarantee
d) Empowering the Panchayats to formulate, implement, operate and maintain drinking water
Projects
Correct Answer: B
Your Answer: Unanswered

IASbaba
Web: http://ilp.iasbaba.com/ Score:
Email: ilp@iasbaba.com 0.00 / 200
Page 85
Set 3 Block 1:
Exam Title :
Geography & ...
Email : misrapulkit@yahoo.in
Contact :
Explanation

Solution (b)

The PM Gramin Digital Saksharta Abhiyan (PMGDISHA) aimed at imparting digital literacy to
citizens in rural areas.

PMGDISHA is government initiative with an objective to impart digital literacy among the
people residing in rural areas. It is an important initiative under Modi’s vision of ‘Digital India’
that intends at making one person in every family digitally literate.

This scheme will focus on making at last six crore people in rural areas (across States/UTs)
digitally literate. By March 31, 2019, it is expected to reach around 40 per cent rural
households in the country.

Under free of cost PMGDISHA scheme, people in the rural area will be trained to operate a
computer, tablet, smartphones, etc. They will be taught how to access the Internet, government
services, undertake digital payment, compose e-mails, etc.

The citizens of rural India will be taught to use applications related to digital payments so they
can participate in the process of nation-building. This scheme will help to connect the digital
divide.

The marginalised sections of society like Scheduled Castes (SC)/Scheduled Tribes (ST),
Minorities, Below Poverty Line (BPL), differently-abled, all will be a part of this scheme. The
scheme aims to empower women in the rural India.

Illiterate people between the age group 14-60 years, nominated from every eligible rural
household, are qualified for the scheme.

Source: http://pib.nic.in/newsite/PrintRelease.aspx?relid=171446

QUESTION 69.
Consider the following statements about Solar Energy Corporation of India (SECI)

1. It is under the Ministry of Power

2. It is the designated agency for implementation of this wind power scheme

Select the correct statements

a) 1 Only
b) 2 Only
c) Both 1 and 2
d) Neither 1 nor 2
Correct Answer: B
Your Answer: Unanswered
Explanation

Solution (b)

Solar Energy Corporation of India Ltd. (SECI) is a company of the Ministry of New and
Renewable Energy, Government of India, established to facilitate the implementation of

IASbaba
Web: http://ilp.iasbaba.com/ Score:
Email: ilp@iasbaba.com 0.00 / 200
Page 86
Set 3 Block 1:
Exam Title :
Geography & ...
Email : misrapulkit@yahoo.in
Contact :
Jawaharlal Nehru National Solar Mission.[3] It is the only public sector undertaking dedicated
to the solar energy sector. The company's mandate has been broadened to cover the entire
renewable energy domain and the company will be renamed to Renewable Energy Corporation
of India (RECI).

It is the designated agency for implementation of this wind power scheme. Power Trading
Corporation will buy the power after entering into power purchase agreements with the
successful bidders. The responsibility for the inter-state transmission system connectivity and
long- term access lies with the wind project developer.

Source: http://www.business-standard.com/article/economy-policy/wind-power-tariff-
fall-to-historic-low-of-rs-2-64-unit-117100500228_1.html

QUESTION 70.
Consider the following statements about ‘ Vishishtadvaita ’

1. It is one of the most popular schools of the Vaisheshika school of Hindu philosophy

2. Ramanuja is the main proponent of Vishishtadvaita philosophy

Select the correct statements

a) 1 Only
b) 2 Only
c) Both 1 and 2
d) Neither 1 nor 2
Correct Answer: B
Your Answer: Unanswered
Explanation

Solution (b)

Vishishtadvaita is one of the most popular schools of the Vedanta school of Hindu philosophy.
Vedanta literally means the end of the Vedas. VishishtAdvaita (literally "Advaita with
uniqueness; qualifications") is a non-dualistic school of Vedanta philosophy. It is non-dualism of
the qualified whole, in which Brahman alone exists, but is characterized by multiplicity. It can
be described as qualified monism or qualified non-dualism or attributive monism. It is a school
of Vedanta philosophy which believes in all diversity subsuming to an underlying unity.

Ramanuja, the main proponent of Vishishtadvaita philosophy contends that the Prasthanatrayi
("The three courses"), namely the Upanishads, the Bhagavad Gita, and the Brahma Sutras are
to be interpreted in a way that shows this unity in diversity, for any other way would violate
their consistency.

Source: 1000th birth anniversary

QUESTION 71.
Which of the following is/are correctly matched?

BPM (Border Personnel Meeting) points State/Region

IASbaba
Web: http://ilp.iasbaba.com/ Score:
Email: ilp@iasbaba.com 0.00 / 200
Page 87
Set 3 Block 1:
Exam Title :
Geography & ...
Email : misrapulkit@yahoo.in
Contact :

1. Kibithu - Ladakh

2. Bum La - Arunachal Pradesh

3. Daulat Beg Oldi - Sikkim

Select the correct code:

a) 1 and 2
b) Only 2
c) 1 and 3
d) 2 and 3
Correct Answer: B
Your Answer: Unanswered
Explanation

Solution (b)

There are five BPM points along the Line of Actual Control (LAC) at Bum La and Kibithu in
Arunachal Pradesh, Daulat Beg Oldi and Chushul in Ladakh, and Nathu La in Sikkim.

Source: http://www.thehindu.com/news/national/no-bpm-between-india-china/
article19781548.ece

QUESTION 72.
WiMAX is a

a) Broadband wireless technology


b) Self-Driving Vehicles
c) Quantum Computer
d) 3D Printing and Artificial Intelligence
Correct Answer: A
Your Answer: Unanswered
Explanation

Solution (a)

Worldwide Interoperability for Microwave Access

• It is a wireless industry coalition dedicated to the advancement of IEEE 802.16 standards


for broadband wireless access (BWA) networks.
• WiMAX can provide at-home or mobile Internet access across whole cities or countries. In
many cases this has resulted in competition in markets which typically only had access
through an existing incumbent DSL (or similar) operator.
• Additionally, given the relatively low costs associated with the deployment of a WiMAX
network (in comparison with 3G, HSDPA, xDSL, HFC or FTTx), it is now economically
viable to provide last-mile broadband Internet access in remote locations.
• WiMAX is competing with the 3rd Generation Partnership Project (3GPP)'s Long-Term
Evolution (LTE) in the 4G market.

IASbaba
Web: http://ilp.iasbaba.com/ Score:
Email: ilp@iasbaba.com 0.00 / 200
Page 88
Set 3 Block 1:
Exam Title :
Geography & ...
Email : misrapulkit@yahoo.in
Contact :

Source: http://www.thehindu.com/sci-tech/science/niist-team-fabricates-a-wearable-
antenna/article19778167.ece

QUESTION 73.
Which of the following is/are correctly matched?

1. Kambala – Andhra Pradesh

2. Jalikattu – Tamil Nadu

3. Aanaval Pidi – Kerala

Select the correct code:

a) 1 and 2
b) 2 and 3
c) 1 and 3
d) All of the above
Correct Answer: B
Your Answer: Unanswered
Explanation

Solution (b)

· Kambala – Karnataka

· Jalikattu – Tamil Nadu

· Aanaval Pidi (Catching elephant’s tail) – Kerala

Kambala is an annual buffalo race which is a tradition in the Karnataka's Dakshina Kannada and
Udupi districts' farming community. This area is called Tulunadu (Land of Tulu Language).

A pair of buffaloes are tied to the plough and one person anchors it, beating the buffaloes with a
stick to run faster. There are two parallel muddy tracks, on which two competing pairs of
buffaloes run. Fastest team wins.

Source: http://www.thehindu.com/news/national/karnataka/kambala-racers-to-get-
insurance-cover/article19781797.ece

QUESTION 74.
Consider the following statements about National Wildlife Action Plan (NWAP) 2017-2031

1. It was drafted by a 12-member committee chaired by JC Kala

2. It underlined an increased role of private sector in wildlife conservation

IASbaba
Web: http://ilp.iasbaba.com/ Score:
Email: ilp@iasbaba.com 0.00 / 200
Page 89
Set 3 Block 1:
Exam Title :
Geography & ...
Email : misrapulkit@yahoo.in
Contact :

3. It recognises the concerns relating to climate change impact on wildlife and stressed on
integrating actions that need to be taken for its mitigation and adaptation into wildlife
management planning processes

Select the correct statements

a) 1 and 2
b) 2 and 3
c) 1 and 3
d) All of the above
Correct Answer: D
Your Answer: Unanswered
Explanation

Solution (d)

National Wildlife Action Plan (NWAP) 2017-2031

About

• The third action plan comes after the first plan in 1983 and second from 2002 till
2016 .
• This is the first time India has recognised the concerns relating to climate change
impact on wildlife and stressed on integrating actions that need to be taken for its
mitigation and adaptation into wildlife management planning processes.
• The plan was initiated in February 2016 by environment ministry. This plan was drafted by
a 12-member committee chaired by JC Kala , a former secretary to the ministry.
• The plan adopts a "landscape approach" in conservation of all wildlife - uncultivated flora
and fauna - that have an ecological value to the ecosystem and to mankind irrespective of
where they occur.
• It calls to make people an intrinsic part of the process to check rising human-animal
conflict
• It gives special emphasis to recovery to threatened species of wildlife while conserving
their habitats.
• It also underscores implementation of endangered species recovery plan of wild animals
in all ecosystems --- terrestrial, inland aquatic, costal and marine.
• It underlined an increased role of private sector in wildlife conservation.
• The plan lays down that the Centre would ensure that adequate and sustained funding
including Corporate Social Responsibility funds are made available for the National
Wildlife Action Plan implementation.

Source: http://pib.nic.in/newsite/PrintRelease.aspx?relid=171329

QUESTION 75.
Consider the following statements about Global Wildlife Programme (GWP)

1. It is led by the World Bank

2. It is funded by the Global Environment Facility

3. It seeks to address the illegal wildlife trade (IWT) countries in Asia, South America and Africa

IASbaba
Web: http://ilp.iasbaba.com/ Score:
Email: ilp@iasbaba.com 0.00 / 200
Page 90
Set 3 Block 1:
Exam Title :
Geography & ...
Email : misrapulkit@yahoo.in
Contact :

Select the correct statements

a) 1 and 2
b) 2 and 3
c) 1 and 3
d) All of the above
Correct Answer: A
Your Answer: Unanswered
Explanation

Solution (a)

Launched in 2015, the Global Wildlife Program (GWP)—A Global Partnership on Wildlife
Conservation and Crime Prevention for Sustainable Development—is a $131 million grant
program funded by the Global Environment Facility (GEF) and led by the World Bank Group.
The GWP seeks to address the illegal wildlife trade (IWT) across 19 countries in Asia and Africa
by serving as a platform for knowledge exchange and coordination, and supporting on-the-
ground actions.

Combating the illegal trade in wildlife is a high priority for the GEF—in their sixth cycle of
funding they created a new strategic objective to prevent the extinction of threatened species
within their Biodiversity Strategy. The GWP targets this objective as well as others within the
GEF’s focal areas of land degradation, climate change and sustainable forest management.

The reach of the GWP allows the program to achieve greater impact than if national projects
worked independently, accelerate the sharing of best practices and lessons learned, and
coordinate outreach with partners, collaborators and donors.

Threats to wildlife can be significantly reduced through a concerted effort to deploy tools and
resources along the entire IWT supply chain. The GWP does this by reducing poaching at the
site level through the engagement of local communities and by conserving and protecting
wildlife natural habitats; controlling wildlife crime and reducing trafficking through effective
law enforcement; and reducing demand for wildlife by raising awareness and changing
behaviour.

Each of the 20 GWP projects tackles one or more of these components across this supply chain.

Source: India jointly hosted the Global Wildlife Programme (GWP) with World Bank
and United Nations Development Programme.

QUESTION 76.
Which of the following are members of Friends of Fish ( FoFs )?

1. India

2. New Zealand

3. Pakistan

4. US

Select the correct code:

IASbaba
Web: http://ilp.iasbaba.com/ Score:
Email: ilp@iasbaba.com 0.00 / 200
Page 91
Set 3 Block 1:
Exam Title :
Geography & ...
Email : misrapulkit@yahoo.in
Contact :

a) 1, 2 and 3
b) 2, 3 and 4
c) 1, 3 and 4
d) 1, 2 and 4
Correct Answer: B
Your Answer: Unanswered
Explanation

Solution (b)

Friends of Fish (FoFs)

About

• Issues: Rules (subsidies)


• Informal coalition seeking to significantly reduce fisheries subsidies. From time to time
other WTO members also identify themselves as “Friends of Fish”“
• WTO members (11): Argentina, Australia, Chile, Colombia, Ecuador, Iceland, New
Zealand, Norway, Pakistan, Peru, US

Source: http://www.thehindubusinessline.com/economy/policy/wto-talks-india-
proposes-scrapping-subsidies-for-fishing-only-outside-territorial-waters/
article9883977.ece

QUESTION 77.
National Waterway 4 (NW-4) connects which of the following states?

1. Telangana

2. Kerala

3. Andhra Pradesh

4. Tamil Nadu

Select the correct code:

a) 1, 2 and 3
b) 2, 3 and 4
c) 1, 3 and 4
d) All of the above
Correct Answer: C
Your Answer: Unanswered
Explanation

Solution (c)

National Waterway 4 (NW-4)

IASbaba
Web: http://ilp.iasbaba.com/ Score:
Email: ilp@iasbaba.com 0.00 / 200
Page 92
Set 3 Block 1:
Exam Title :
Geography & ...
Email : misrapulkit@yahoo.in
Contact :

About

• It has been declared as an Indian National Waterway and is currently under development.
It connects the Indian states of Telangana, Andhra Pradesh, Tamil Nadu, and the union
territory of Puducherry.
• The NW-4 would be developed in three phases. In the first phase, the 82 km stretch from
Muktyala to Vijayawada will be developed.
• This would be followed by the second phase where 233 km will be developed from
Vijayawada to Kakinada and Rajahmundry to Polavaram on Godavari river.
• The third phase will connect Commamur canal, Buckingham canal and the balance
stretches of river Krishna and Godavari across 573 km.
• The first phase involves development of a water channel from Muktyala to Vijayawada on
Krishna river along with four floating terminals and three fixed terminals to handle cargo
operations.

IASbaba
Web: http://ilp.iasbaba.com/ Score:
Email: ilp@iasbaba.com 0.00 / 200
Page 93
Set 3 Block 1:
Exam Title :
Geography & ...
Email : misrapulkit@yahoo.in
Contact :

IASbaba
Web: http://ilp.iasbaba.com/ Score:
Email: ilp@iasbaba.com 0.00 / 200
Page 94
Set 3 Block 1:
Exam Title :
Geography & ...
Email : misrapulkit@yahoo.in
Contact :

Source: http://www.business-standard.com/article/pti-stories/vp-naidu-lays-foundation-
stone-for-inland-waterway-in-ap-117100300417_1.html

QUESTION 78.
Which of the following countries are members of Paris Club?

1. China

2. Japan

3. South Korea

4. Israel

Select the correct code:

a) 1, 3 and 4
b) 2, 3 and 4
c) 1, 2 and 4
d) 1, 2 and 3
Correct Answer: B
Your Answer: Unanswered
Explanation

Solution (b)

Paris Club

About

• An informal group of creditor nations whose objective is to find workable solutions to


payment problems faced by debtor nations.
• The Paris Club has 19 permanent members, including most of the western European and
Scandinavian nations, the United States of America, the United Kingdom and Japan.
• It is similar to the London club, which is a group of commercial bankers formed in 1976 to
deal with the financial problems of Zaire, and is focussed on providing various forms of
debt relief to countries that face financial distress due to their heavy debt load.
• The Paris Club stresses the informal nature of its existence and deems itself a "non-
institution."
• As an informal group, it has no official statutes and no formal inception date, although its
first meeting with a debtor nation was in 1956, with Argentina.

The following countries are permanent Paris Club members:

· Australia

· Austria

· Belgium

· Brazil

IASbaba
Web: http://ilp.iasbaba.com/ Score:
Email: ilp@iasbaba.com 0.00 / 200
Page 95
Set 3 Block 1:
Exam Title :
Geography & ...
Email : misrapulkit@yahoo.in
Contact :

· Canada

· Denmark

· Finland

· France

· Germany

· Ireland

· Israel

· Italy

· Japan

· Korea

· Netherlands

· Norway

· Russian Federation

· Spain

· Sweden

· Switzerland

· United Kingdom

· United States Of America

Source: http://www.thehindu.com/opinion/op-ed/what-is-paris-club-in-economics/
article19803224.ece

QUESTION 79.
The Square Kilometre Array (SKA) is an international project to build

a) A Mars Rover
b) Mars Orbiter
c) Space Junk Cleaner
d) None of the above
Correct Answer: D
Your Answer: Unanswered
Explanation

Solution (d)

Square Kilometre Array (SKA)

IASbaba
Web: http://ilp.iasbaba.com/ Score:
Email: ilp@iasbaba.com 0.00 / 200
Page 96
Set 3 Block 1:
Exam Title :
Geography & ...
Email : misrapulkit@yahoo.in
Contact :

About

• This reflects the original desire to construct a telescope with up to one square kilometre
in collecting surface through an array of antennas distributed over a much larger area.
• The Square Kilometre Array (SKA) is an international project to build a radio telescope
tens of times more sensitive and hundreds of times faster at mapping the sky than today’s
best radio astronomy facilities. Simply put: the world’s largest radio telescope.
• The SKA is not a single telescope, but a collection of various types of antennas, called an
array, to be spread over long distances.

QUESTION 80.
Consider the following statements about ImTeCHO

1. It is an innovative mobile phone application to improve performance of ASHAs through better


supervision, support and motivation for increasing coverage of proven maternal, new-born and
child health interventions among resource-poor settings

2. It stands for Innovative mobile-phone Technology for Community Health Operations’

Select the correct statements

a) 1 Only
b) 2 Only
c) Both 1 and 2
d) Neither 1 nor 2
Correct Answer: C
Your Answer: Unanswered
Explanation

Solution (c)

ImTeCHO

Innovative Mobile Phone Technology for Community Health Operation (ImTeCHO)

• It is an innovative mobile phone application to improve performance of ASHAs through


better supervision, support and motivation for increasing coverage of proven maternal,
newborn and child health interventions among resource-poor settings in India.
• ImTeCHO provides support to ASHAs throughout continuum of care starting from
beginning of pregnancy up to child completes critical first two years of age.
• ImTeCHO is used every day by ASHAs for checking their daily schedule, guide home
visitations, record services provided on monthly health and nutrition day, track high risk
cases, and record births and deaths.
• Web interface can be used every day for tracking high risk cases, obtain information about
critical indicators, and manage incentives and supplies.

ImTeCHO has two components:

• Mobile phone application which is used by ASHAs and


• Web interface (computer) which is used by PHC staff including medical officers.

IASbaba
Web: http://ilp.iasbaba.com/ Score:
Email: ilp@iasbaba.com 0.00 / 200
Page 97
Set 3 Block 1:
Exam Title :
Geography & ...
Email : misrapulkit@yahoo.in
Contact :

Source: http://indianexpress.com/article/india/narendra-modi-gujarat-visit-live-
updates-vadnagar-hatkeshwar-temple-antyodaya-express-indradhanush-imtecho-
bharuch-4879943/

QUESTION 81.
Consider the following rivers:

1. Amravati

2. Koyna

3. Kabini

4. Bhadra

Which of the above is/are tributaries of Cauvery ( Kaveri ) river?

a) 2 and 4 only
b) 1, 2 and 3 only
c) 3 only
d) 1 and 3 only
Correct Answer: D
Your Answer: Unanswered
Explanation

Solution (d)

The Kaveri basin is estimated to 72,000 km2 with many tributaries including the Shimsha, the
Hemavati, the Arkavati, Honnuhole, Lakshmana Tirtha, Kabini, Bhavani River, the Lokapavani,
the Noyyal and the Amaravati River.

Koyna is the tributary of Krishna River whereas Bhadra is the tributary of Tungabhadra.

Tributaries

· Amaravati, Arkavathy, Bhavani, Chinnar, Hemavati, Honnuhole, Kabini, Kannika, Kollidam,


Lakshmana Tirtha, Lokapavani, Noyyal, Pambar, Shimsha, Sujyothi

Riparian States and UTs

· Karnataka, Kerala, Pondicherry, Tamil Nadu

QUESTION 82.
Match the India's mountain passes with their location in the respective states.

Pass State

1. Nathu la Sikkim

IASbaba
Web: http://ilp.iasbaba.com/ Score:
Email: ilp@iasbaba.com 0.00 / 200
Page 98
Set 3 Block 1:
Exam Title :
Geography & ...
Email : misrapulkit@yahoo.in
Contact :

2. Bhor Ghat Madhya Pradesh

3. Pal Ghat Kerala

4. Lipulekh Himanchal Pradesh

Which of the above given pair/s is/are correctly matched?

a) 1 and 4 only
b) 2 and 3 only
c) 3 and 4 only
d) 1 and 3 only
Correct Answer: D
Your Answer: Unanswered
Explanation

Solution (d)

Nathu La is a mountain pass in the Himalayas. It connects the Indian state of Sikkim with
China's Tibet Autonomous Region.

Bhor Ghat or Bor Ghat, Bhore Ghaut, is a mountain passage located between Palasdari and
Khandala in Maharashtra , India

Palakkad Gap or Palghat Gap is a low mountain pass in the Western Ghats between
Coimbatore in Tamil Nadu and Palakkad in Kerala

Lipulekh (also known as Tri-Corner) is a Himalayan pass between India and China near
their tripoint with Nepal connecting the North Western Cornered Byash Valley of Nepal and
Indian State of Uttarakhand with the old trading town of Taklakot (Purang) in Tibet.

QUESTION 83.
What is the correct sequence of Capitals of India States as one proceeds from north to south?

1) Gangtok

2) Itanagar

3) Kohima

4) Shillong

Consider the following options and choose the correct sequence.

a) 1-2-3-4
b) 2-1-3-4
c) 1-2-4-3
d) 2-1-4-3
Correct Answer: A
Your Answer: Unanswered

IASbaba
Web: http://ilp.iasbaba.com/ Score:
Email: ilp@iasbaba.com 0.00 / 200
Page 99
Set 3 Block 1:
Exam Title :
Geography & ...
Email : misrapulkit@yahoo.in
Contact :
Explanation

Solution (a)

QUESTION 84.
Which of the following pairs is/are correctly matched?

Dam/Project River

1. Koyna Kaveri

2. Tehri Alaknanda

3. Bisalpur Banas

Select the correct answer using the code given below.

a) 1 and 2 only
b) 2 only
c) 1 and 3 only
d) 3 only
Correct Answer: D
Your Answer: Unanswered
Explanation

Solution (d)

Tehri dam – Bhagirathi River near Tehri in Uttarakhand, India.

Koyna dam – koyna river

IASbaba
Web: http://ilp.iasbaba.com/ Score:
Email: ilp@iasbaba.com 0.00 / 200
Page 100
Set 3 Block 1:
Exam Title :
Geography & ...
Email : misrapulkit@yahoo.in
Contact :

Bisalpur – Banas river. The Banas is a river of Rajasthan state in western India. It is a tributary
of the Chambal River, which in turn flows into the Yamuna, a tributary of the Ganges. The Banas
is approximately 512 kilometres in length. It is also known as 'Van Ki Asha' (Hope of forest).
Bisalpur Dam is a gravity dam on the Banas River near Deoli in Tonk district, Rajasthan, India.

QUESTION 85.
What is the correct sequence of occurrence of following mountains ranges of south India as one
proceeds from North and South?

1. Palkonda Hills

2. Javadi Hills

3. Sirumali Hills

4. Palani Hills

Select the correct answer using the code given below.

a) 1-2-3-4
b) 1-2-4-3
c) 2-1-3-4
d) 2-3-1-4
Correct Answer: A
Your Answer: Unanswered
Explanation

Solution (a)

Palconda Hills are in the north followed by Javadi, Shevaroy and Palani is located in transverse
mode as southernmost hills.

IASbaba
Web: http://ilp.iasbaba.com/ Score:
Email: ilp@iasbaba.com 0.00 / 200
Page 101
Set 3 Block 1:
Exam Title :
Geography & ...
Email : misrapulkit@yahoo.in
Contact :

QUESTION 86.
Choose correct statements regarding vindyanchal range.

1. There is a great boundary fault where Vindyanchal & Aravali ranges meet.

2. Mahadeo and Maikal mountains form a major part of this range.

3. The Vindhyan range is bounded by the Central Highlands on the south and the Aravalis on
the northwest

Select the appropriate option:

a) 1 and 2 only
b) 1 and 3 only
c) 2 and 3 only
d) All of them are correct.
Correct Answer: B
Your Answer: Unanswered
Explanation

Solution (b)

The Vindhyan range is bounded by the Central Highlands on the south and the Aravalis on the
northwest. It extends from Jobat (Gujarat) and Chittorgarh (Rajasthan) to Sasaram in Bihar for
about 1050 km with general elevation between 450 to 600 metres. The western end of the
Vindhyan range is in Gujarat at the eastern side of the Gujarat peninsula, near the border with

IASbaba
Web: http://ilp.iasbaba.com/ Score:
Email: ilp@iasbaba.com 0.00 / 200
Page 102
Set 3 Block 1:
Exam Title :
Geography & ...
Email : misrapulkit@yahoo.in
Contact :
Rajasthan and Madhya Pradesh. Reaching the sub-continent proper, the range runs east and
north nearly to the Ganges River at Mirzapur. The area to the north and west of the range are
arid and inhospitable, located in the shadow of both the Vindhya and the higher Aravalli range
to the south blocking the prevailing winds. The southern slopes of the Vindhyan Range are
drained by the Narmada River, which proceeds westward to the Arabian Sea in the wide valley
between the Vindhya Range and the parallel Satpura Range farther to the south. The northern
slopes of the range are drained by tributaries of the Ganges, including the Kali Sindh, Parbati,
Betwa, & Ken (both are tributary of the Yamuna, ), Son & Tamsa or Tons both are tributary of
the Ganges, drains the southern slopes of the range at its eastern end.

Do you know?

· The Great Boundary Fault (GBF) separates the Aravallis from the Vindhyan Mountains.

· The Maikal Hills are an eastern part of the Satpuras whereas The Mahadeo Hills are a range
of hills in Madhya Pradesh state of central India. The hills form the central part of the Satpura
Range.

IASbaba
Web: http://ilp.iasbaba.com/ Score:
Email: ilp@iasbaba.com 0.00 / 200
Page 103
Set 3 Block 1:
Exam Title :
Geography & ...
Email : misrapulkit@yahoo.in
Contact :

QUESTION 87.

IASbaba
Web: http://ilp.iasbaba.com/ Score:
Email: ilp@iasbaba.com 0.00 / 200
Page 104
Set 3 Block 1:
Exam Title :
Geography & ...
Email : misrapulkit@yahoo.in
Contact :

Which of the following Indian states make a boundary with Bangladesh?

1. Assam

2. West Bengal

3. Jharkhand

4. Mizoram

5. Sikkim

6. Meghalaya

Select the correct option

a) 1, 2, 4 and 6 only
b) 1, 2, 4, 5 and 6 only
c) 1, 2, 3 and 6 only
d) 1, 2, 4 and 5 only
Correct Answer: A
Your Answer: Unanswered
Explanation

Solution (a)

IASbaba
Web: http://ilp.iasbaba.com/ Score:
Email: ilp@iasbaba.com 0.00 / 200
Page 105
Set 3 Block 1:
Exam Title :
Geography & ...
Email : misrapulkit@yahoo.in
Contact :

QUESTION 88.
Cherrapunji and Mawsynram of Meghalaya which receives highest rainfall are in

a) Garo hills
b) Khasi Hills
c) Jaintia Hills
d) None of the above

IASbaba
Web: http://ilp.iasbaba.com/ Score:
Email: ilp@iasbaba.com 0.00 / 200
Page 106
Set 3 Block 1:
Exam Title :
Geography & ...
Email : misrapulkit@yahoo.in
Contact :
Correct Answer: B
Your Answer: Unanswered
Explanation

Solution (b)

Mawsynram is located at 25° 18′ N, 91° 35′ E, at an altitude of about 1,400 metres (4,600 ft), 16
km west of Cherrapunji, in the Khasi Hills in the state of Meghalaya (India)

QUESTION 89.
Which of the following river/s flows through a rift valley?

1. Damodar

2. Tapi

3. Mahanadi

4. Luni

Select the correct option

a) 1 and 2 only
b) 1 and 3 only
c) 2 and 4 only
d) 1,2 and 3 only
Correct Answer: A
Your Answer: Unanswered
Explanation

Solution (a)

Apart from Narmada, Damodar and Tapi also flow through a rift valley.

Narmada

It is one of only three major rivers in peninsular India that run from east to west (longest west
flowing river), along with the Tapti River and the Mahi River.

It is one of the rivers in India that flows in a rift valley, flowing west between
the Satpura and Vindhya ranges. The other rivers which flow through rift valley
include Damodar River in Chota Nagpur Plateau and Tapti. The Tapti River and Mahi River also
flow through rift valleys, but between different ranges.

QUESTION 90.
Consider the following statements regarding “ Dooars ” or “ Duar ”

1. Dooars are the feature of Uttarakhand Himalayas

2. It has also been used for Tea plantations

IASbaba
Web: http://ilp.iasbaba.com/ Score:
Email: ilp@iasbaba.com 0.00 / 200
Page 107
Set 3 Block 1:
Exam Title :
Geography & ...
Email : misrapulkit@yahoo.in
Contact :

Which of the above statement(s) is/are INCORRECT?

a) 1 only
b) 2 only
c) Both
d) None
Correct Answer: A
Your Answer: Unanswered
Explanation

Solution (a)

The Dooars or Duars are the alluvial floodplains in northeastern India that lie south of the outer
foothills of the Himalayas and north of the Brahmaputra River basin.

In place of the Shiwaliks in the Darjeeling and Sikkim Himalayas the duar formations are
important, which have also been used for the development of tea gardens.

QUESTION 91.
Consider the following statements

1. It demarcates Nepal’s western border with India.

2. It is a tributary of Ghaghra

3. It makes a boundary of Pilibhit tiger reserve

Which of the following river is described above?

a) Gandak
b) Kali
c) Kosi
d) Mohana
Correct Answer: B
Your Answer: Unanswered
Explanation

Solution (b)

River Kali makes a boundary line between India and Nepal.

A treaty had been signed between India and Nepal for developing Hydro power project on river
Kali or Sarda or Mahakali etc. The Treaty on Integrated Development of Mahakali River was
signed by the Prime Minister of His Majesty's Government of Nepal and the Prime Minister of
the Government of India in February 1996 and which came into effect in June 1997.

It concerns with the Integrated Development of the Mahakali (Sharda in India) River including
Sharda Barrage (existing), Tanakpur Barrage (existing) and Pancheshwar Project (Proposed –
under Planning). Pancheshwar Multipurpose Project (PMP) on the river Mahakali is the
centerpiece of the Treaty.

IASbaba
Web: http://ilp.iasbaba.com/ Score:
Email: ilp@iasbaba.com 0.00 / 200
Page 108
Set 3 Block 1:
Exam Title :
Geography & ...
Email : misrapulkit@yahoo.in
Contact :

QUESTION 92.
Shale gas reserves are found in which of the following areas?

1) Cauvery basin

2) Krishna-Godavari basin

3) Assam- Arakan

Select the correct answer using the code

a) 1 and 2 only
b) 2 and 3 only
c) 1 and 2 only
d) All of them are correct.
Correct Answer: D
Your Answer: Unanswered
Explanation

Solution (d)

· Shale gas is a form of natural gas that is found trapped within pores of sedimentary shale
rocks beneath the surface of the earth.

· India holds an estimated 527 trillion cubic feet (tcf) of shale gas reserves. The Ministry of
Petroleum and Natural Gas (MoPNG) has identified six basins as potentially shale gas bearing.
These are Cambay, Assam-Arakan, Gondwana, Krishna-Godavari, Kaveri, and the Indo-
Gangetic plain . Jambusar located near Vadodara in Gujarat is India ‘s first shale gas
exploratory successfully drilled in Cambay region.

QUESTION 93.
Consider the following statements about the Aravallis

1. They are the oldest mountain ranges of India.

2. Aravalli is an example of Block Mountains.

3. Guru Shikhar is the highest peak of Aravalli range.

4. Aravalli hills lie perpendicular to the direction of monsoon winds causing heavy rainfall in
eastern Rajasthan.

Which of the above statements are correct?

a) 1 and 3
b) 2 and 4
c) 1,2 and 3
d) All of the above
Correct Answer: A

IASbaba
Web: http://ilp.iasbaba.com/ Score:
Email: ilp@iasbaba.com 0.00 / 200
Page 109
Set 3 Block 1:
Exam Title :
Geography & ...
Email : misrapulkit@yahoo.in
Contact :
Your Answer: Unanswered
Explanation

Solution (a)

Aravalli range is an old folded mountain range. They lie parallel to the monsoon winds causing
no interception hence very little rainfall.

QUESTION 94.
Consider the following statements regarding Bhabhar plains:

1. It is the northern most tracts of Indian plains lying at the foothills of Siwaliks.

2. It contains fine alluvium deposited by the Himalayan streams.

3. It is very fertile and suitable for cultivation of crops.

Which of the above statements is/are INCORRECT?

a) All of the above


b) 1 only
c) 2 and 3
d) None of the above
Correct Answer: C
Your Answer: Unanswered
Explanation

Solution (c)

Bhabar region lies along the foot of the Siwaliks from the Indus to the Tista. It comprises of
pebble-studded rocks in the shape of porous beds. It is due to porosity of the rocks; the streams
disappear and flow underground. The underground streams of the Bhabar re-emerge on the
surface and give birth to marshy area. The region is not much suitable for agriculture.

QUESTION 95.
Consider the statements w.r.t Ganges Drainage System and Peninsular Drainage System

1. Peninsular rivers bear the exception of flowing through Rift valleys, like Narmada, while
Ganges Rivers have no such feature.

2. All Peninsular rivers are characterized by fixed course, absence of meanders and non-
perennial flow of water while Ganges rivers are perennial, having meanders and have non-fixed
courses.

Select the correct statement/s

a) Only 1
b) Only 2
c) Both

IASbaba
Web: http://ilp.iasbaba.com/ Score:
Email: ilp@iasbaba.com 0.00 / 200
Page 110
Set 3 Block 1:
Exam Title :
Geography & ...
Email : misrapulkit@yahoo.in
Contact :

d) None
Correct Answer: D
Your Answer: Unanswered
Explanation

Solution (d)

Damodar River, a river in Ganges River System occupies the eastern margins of the
Chotanagpur Plateau where it flows through a rift valley. So, first statement is wrong.

Peninsular rivers are characterized by fixed course, absence of meanders and non-perennial
flow of water. The Narmada and Tapi which flow through the rift valley are, however,
exceptions. Hence none of the statements are correct.

QUESTION 96.
Consider the following:

1. Cardomom Hills Coromandal coast

2. Kaimur Hills Konkan Coast

3. Mahadeo Hills Central India

4. Mikir Hills North- East India

Which of the above are correctly matched?

a) 1 and 2
b) 2 and 3
c) 3 and 4
d) 2 and 4
Correct Answer: C
Your Answer: Unanswered
Explanation

Solution (c)

The Coromandel Coast is the southeastern coast region of the Indian Subcontinent, between
the Eastern Ghats and the Bay of Bengal of the Indian Ocean.

The coastline runs between False Divi Point in the north to Kanyakumari in the south. Its
definition can also include the northwestern coast of the island of Sri Lanka

Cardomom Hills - Western Ghats

Kaimur Range is the eastern portion of the Vindhya Range, about 483 kilometres (300 mi)
long, extending from around Katangi in Jabalpur district of Madhya Pradesh to
around Sasaram in Rohtas district of Bihar.

It passes through the Rewa and Mirzapur divisions. The range never rises more than a few
hundred metres above the surrounding plains and has a maximum width of around 80 km

IASbaba
Web: http://ilp.iasbaba.com/ Score:
Email: ilp@iasbaba.com 0.00 / 200
Page 111
Set 3 Block 1:
Exam Title :
Geography & ...
Email : misrapulkit@yahoo.in
Contact :

Konkan, also known as the Konkan Coast or Kokan, is a rugged section of the western
coastline of India. It is a 720 kilometres (450 miles) long coastline. It consists of the coastal
districts of the western Indian states of Maharashtra, Goa, and Karnataka.

The ancient Saptakonkana is a slightly larger region. The region is known as Karavali in
Karnataka.

Mikir Hills are a group of hills located to the south of the Kaziranga National Park, Assam. It is
part of the Karbi Anglong Plateau.

The Mahadeo Hills are a range of hills in Madhya Pradesh state of central India. The hills are
situated in the northern section of the Satpura Range.

QUESTION 97.
Which of the following channels is the National Waterway 3?

a) Allahabad to Haldia stretch of Ganga.


b) Dubri -Sadia Stretch of Brahmaputra.
c) Kottapuram -Kollam stretch of West Canal
d) Lakhipur to Bhanga stretch of Barak river
Correct Answer: C
Your Answer: Unanswered
Explanation

Solution (c)

National
Extent Kilometer River
Waterways

Ganga-Bhagirathi-
NW 1 Allahabad-Haldia stretch 1620km
Hoogly

NW 2 Sadiya-Dhubri stretch 891km Brahmaputra

NW3 Kottapuram-Kollam stretch 205km West Coast Canal

Kakinada Puducherry
Canal
NW 4 1995km Krishna, Godavari
+ Godavari + Krishna

NW 5 Talcher Dhamra 585km Brahmani

IASbaba
Web: http://ilp.iasbaba.com/ Score:
Email: ilp@iasbaba.com 0.00 / 200
Page 112
Set 3 Block 1:
Exam Title :
Geography & ...
Email : misrapulkit@yahoo.in
Contact :

NW 6 Lakhipur to Bhanga 121km Barak

QUESTION 98.
Which of the following statements are correct ‘Radial Drainage Pattern’?

1. They are found when there is a central depression and acts a sink of the streams.

2. The rivers flow in circular fashion around the center .

3. Radial Drainage is found around Amarkantak mountain in India

Select the code from following:

a) 1 and 2
b) 2 and 3
c) 3 only
d) All of the above
Correct Answer: C
Your Answer: Unanswered
Explanation

Solution (c)

Radial drainage pattern:

· Develops around a central elevated point

· Common to conically shaped features as volcanoes

· Rivers originating from the Amarkantak range

IASbaba
Web: http://ilp.iasbaba.com/ Score:
Email: ilp@iasbaba.com 0.00 / 200
Page 113
Set 3 Block 1:
Exam Title :
Geography & ...
Email : misrapulkit@yahoo.in
Contact :

QUESTION 99.
The upland which separates two drainage basins is called:

a) Mountain range
b) Middle Ridge
c) Water divide
d) Watershed
Correct Answer: C
Your Answer: Unanswered
Explanation

Solution (c)

The area drained by a single river system is called a drainage basin. A closer observation on a
map will show that any elevated area such as a mountain or upland, separates two drainage
basins. Such an upland is known as a water divide.

IASbaba
Web: http://ilp.iasbaba.com/ Score:
Email: ilp@iasbaba.com 0.00 / 200
Page 114
Set 3 Block 1:
Exam Title :
Geography & ...
Email : misrapulkit@yahoo.in
Contact :

QUESTION 100.
Arrange the following states in the descending order in terms of its coastline (length)?

a) Gujarat, Andhra Pradesh, Tamil Nadu, Kerala


b) Tamil Nadu, Gujarat, Maharashtra, Karnataka
c) Tamil Nadu, Maharashtra, West Bengal, Karnataka
d) Gujarat, Tamil Nadu, Kerala, Karnataka
Correct Answer: A
Your Answer: Unanswered
Explanation

Solution (a)

The total coastline of India measures about 7,517 km, which is distributed among nine coastal
states and four Union Territories. The nine coastal states are Gujarat, Maharashtra, Goa,
Karnataka, Kerala, Tamil Nadu, Andhra Pradesh, Odisha and West Bengal. The Four coastal
Union Territories are Puducherry, Lakshadweep, Daman & Diu and Andaman & Nicobar Islands.

Gujarat is strategically located with largest share in India’s coastline, followed by Andhra
Pradesh and Tamil Nadu, Kerala and Maharashtra.

IASbaba
Web: http://ilp.iasbaba.com/ Score:
Email: ilp@iasbaba.com 0.00 / 200
Page 115
Set 3 Block 1:
Exam Title :
Geography & ...
Email : misrapulkit@yahoo.in
Contact :

IASbaba
Web: http://ilp.iasbaba.com/ Score:
Email: ilp@iasbaba.com 0.00 / 200
Page 116
Set 3 Block 1:
Exam Title :
Geography & ...
Email : misrapulkit@yahoo.in
Contact :
Review in

IASbaba
Web: http://ilp.iasbaba.com/ Score:
Email: ilp@iasbaba.com 0.00 / 200
Page 117

Potrebbero piacerti anche